62
Editors: Koda-Kimble, Mary Anne; Young, Lloyd Yee; Alldredge, Brian K.; Corelli, Robin L.; Guglielmo, B. Joseph; Kradjan, Wayne A.; Williams, Bradley R. Title: Applied Therapeutics: The Clinical Use Of Drugs, 9th Edition Copyright ©2009 Lippincott Williams & Wilkins > Table of Contents > Section Sixteen - Psychiatric Disorders > Chapter 78 - Schizophrenia Chapter 78 Schizophrenia Rene A. Endow-Eyer Melissa M. Mitchell Jonathan P. Lacro Schizophrenia is a debilitating and emotionally devastating illness with long-term impact on patients' lives. Many experts consider schizophrenia to be the most severe expression of psychopathology, encompassing significant disruptions of thinking, perception, emotion, and behavior. Schizophrenia is usually a lifelong psychiatric disability. Family relationships, social functioning, and employment are frequently affected, and periodic hospitalizations are common. The management of schizophrenia involves multiple strategies to optimize the patient's functional capacity, reduce the frequency and severity of symptom exacerbations, and reduce the overall morbidity and mortality from this disorder. Many patients require comprehensive and continuous care over the course of their lives. The aim of this chapter is to provide a framework for the clinician to develop skills in schizophrenia management by considering the pathophysiology, assessment, clinical course, and treatment of schizophrenia. Epidemiology In the United States, approximately 1% of the population develops schizophrenia during their lifetime. Although schizophrenia affects men and women with equal frequency, there are differences in the age of onset and course of illness. The onset of schizophrenia usually occurs during late adolescence or early adulthood. Studies have shown women have, on average, an onset of schizophrenia 3 to 5 years after men. 1 Women also have their first admission due to psychiatric illness 3 to 6 years after men on average. 1 Prevalence rates are similar throughout the world, but pockets of high prevalence have been reported in some areas. After adjusting for differences in diagnostic criteria for schizophrenia, similar prevalence rates across cultures have been observed. Economic Burden Mental disorders constitute a large part of the global burden of disease, with schizophrenia being among the greatest causes of disability in the United States and the world. 2 Unlike other chronic diseases such as diabetes and hypertension, which occur late in life, schizophrenia usually affects people when they are young followed by a chronic course persisting throughout the patient's lifetime. Most people with schizophrenia experience multiple hospitalizations and generally require social assistance. Even during relatively stable phases of their illness, most people with schizophrenia require support that ranges from a family member to daytime hospitalization. As of 2002, total costs for schizophrenic patients in the United States averaged $62.7 billion annually, taking into account direct and indirect costs, as well as unemployment costs. Direct costs include hospitalization, rehabilitation, professional services, medication, and office visits. Indirect costs include loss of productivity caused by illness, disability, premature death, and economic burden to families. 3 Schizophrenia accounts for approximately 2.5% of total annual health care costs in the United States. Persons afflicted by schizophrenia represent approximately 10% of this country's permanently disabled population and consume 20% of Social Security benefit days. 4,5 In 2004, it was estimated that 70% of patients with schizophrenia receive medications through Medicaid. Because of the advent of newer atypical antipsychotics over the past 20 years, the cost of psychotropic medications now exceeds $10 billion annually. 6 Etiology (Neurobiology) Schizophrenia is a complex disorder with multiple causes. Considering the lack of consistent neuropathology or biomarkers, current theories of the disorder have moved away from the view that it is a single entity, toward a P.78p2 Page 1 of 62 Ovid: Applied Therapeutics: The Clinical Use Of Drugs 17/8/2010 mk:@MSITStore:C:\Users\askari\Desktop\E%20Book\Applied\Applied%20Therapeuti...

Schizophrenia

Embed Size (px)

Citation preview

Page 1: Schizophrenia

Editors: Koda-Kimble, Mary Anne; Young, Lloyd Yee; Alldredge, Brian K.; Corelli, Robin L.; Guglielmo, B.

Joseph; Kradjan, Wayne A.; Williams, Bradley R.

Title: Applied Therapeutics: The Clinical Use Of Drugs, 9th Edition

Copyright ©2009 Lippincott Williams & Wilkins

> Table of Contents > Section Sixteen - Psychiatric Disorders > Chapter 78 - Schizophrenia

Chapter 78

Schizophrenia

Rene A. Endow-Eyer

Melissa M. Mitchell

Jonathan P. Lacro

Schizophrenia is a debilitating and emotionally devastating illness with long-term impact on patients' lives. Many

experts consider schizophrenia to be the most severe expression of psychopathology, encompassing significant

disruptions of thinking, perception, emotion, and behavior. Schizophrenia is usually a lifelong psychiatric disability.

Family relationships, social functioning, and employment are frequently affected, and periodic hospitalizations are

common. The management of schizophrenia involves multiple strategies to optimize the patient's functional

capacity, reduce the frequency and severity of symptom exacerbations, and reduce the overall morbidity and

mortality from this disorder. Many patients require comprehensive and continuous care over the course of their

lives.

The aim of this chapter is to provide a framework for the clinician to develop skills in schizophrenia management by

considering the pathophysiology, assessment, clinical course, and treatment of schizophrenia.

Epidemiology In the United States, approximately 1% of the population develops schizophrenia during their lifetime. Although

schizophrenia affects men and women with equal frequency, there are differences in the age of onset and course of

illness. The onset of schizophrenia usually occurs during late adolescence or early adulthood. Studies have shown

women have, on average, an onset of schizophrenia 3 to 5 years after men.1 Women also have their first admission

due to psychiatric illness 3 to 6 years after men on average.1 Prevalence rates are similar throughout the world, but

pockets of high prevalence have been reported in some areas. After adjusting for differences in diagnostic criteria for schizophrenia, similar prevalence rates across cultures have been observed.

Economic Burden Mental disorders constitute a large part of the global burden of disease, with schizophrenia being among the

greatest causes of disability in the United States and the world.2 Unlike other chronic diseases such as diabetes and

hypertension, which occur late in life, schizophrenia usually affects people when they are young followed by a

chronic course persisting throughout the patient's lifetime. Most people with schizophrenia experience multiple

hospitalizations and generally require social assistance. Even during relatively stable phases of their illness, most

people with schizophrenia require support that ranges from a family member to daytime hospitalization. As of 2002,

total costs for schizophrenic patients in the United States averaged $62.7 billion annually, taking into account direct

and indirect costs, as well as unemployment costs. Direct costs include hospitalization, rehabilitation, professional

services, medication, and office visits. Indirect costs include loss of productivity caused by illness, disability,

premature death, and economic burden to families.3 Schizophrenia accounts for approximately 2.5% of total annual

health care costs in the United States. Persons afflicted by schizophrenia represent approximately 10% of this

country's permanently disabled population and consume 20% of Social Security benefit days.4,5 In 2004, it was

estimated that 70% of patients with schizophrenia receive medications through Medicaid. Because of the advent of

newer atypical antipsychotics over the past 20 years, the cost of psychotropic medications now exceeds $10 billion

annually.6

Etiology (Neurobiology) Schizophrenia is a complex disorder with multiple causes. Considering the lack of consistent neuropathology or

biomarkers, current theories of the disorder have moved away from the view that it is a single entity, toward a

P.78p2

Page 1 of 62Ovid: Applied Therapeutics: The Clinical Use Of Drugs

17/8/2010mk:@MSITStore:C:\Users\askari\Desktop\E%20Book\Applied\Applied%20Therapeuti...

Page 2: Schizophrenia

conceptualization of schizophrenia as a collection of etiologically disparate disorders with common clinical

features.7,8 In this section, the genetic and environmental factors, neuroanatomical, and neurochemical features of

schizophrenia are considered.

Genetic and Environmental Risk Factors First-degree biologic relatives of persons with schizophrenia have a tenfold higher risk of developing the disease

than the general population.9 The risk is highest (40%–50%) in a monozygotic (identical) twin of a person with

schizophrenia.10,11 There is also higher concordance for schizophrenia in monozygotic twins compared with dizygotic

twins, with respective rates being approximately 50% and 15%.12 Because the concordance rate is not 100% in

monozygotic twins or 50% in dizygotic twins, other nongenetic factors must contribute to the development of the

disorder. Environmental factors such as prenatal difficulties (e.g., malnutrition in the first trimester of pregnancy or

influenza in the second trimester), perinatal complications, and various nonspecific stressors may also influence the

development of schizophrenia. Despite the overwhelming evidence that schizophrenia is an inherited illness, a single

“schizophrenia gene” has not been found. Risk for schizophrenia is, to some extent, genetically transmitted and is

probably determined by multiple genes. A review of the molecular genetics of schizophrenia cited linkage studies

implicating chromosomes 6, 8, 10, 13, and 22.13

Neuroanatomy Studies comparing the brains of individuals with schizophrenia with the brains of normal controls have uncovered a

number of important differences. Although these studies suggest that anatomical and functional abnormalities are

associated with schizophrenia, no single pathognomonic abnormality consistently has been found. In addition,

localization of the site(s) responsible for the pathophysiology of schizophrenia remains elusive.

The most consistent structural finding seen with computed tomography and magnetic resonance imaging in patients

with schizophrenia has been enlarged ventricles, particularly the lateral and third ventricles. In addition, there

seems to be a relationship between brain asymmetry and the disease process.14 When unilateral abnormalities have

been reported, they have typically involved the left hemisphere.14 As imaging technology has advanced, magnetic

resonance imaging studies in schizophrenia frequently have shown morphologic abnormalities involving the temporal,

frontal, and parietal lobes, and the subcortical structures.15,16,17,18,19 Findings include decreased neuronal volume

and density, decreased synaptic connections, decreases in synaptophysin (a membrane protein of synaptic vesicles),

and loss of microtubule associated protein and synaptosomal protein in certain regions of the cortex.20,21 The lack of

gliosis in association with the neuroantomical abnormalities in the brains of schizophrenia patients suggests that

these changes occur neurodevelopmentally.22 Functional imaging studies, such as positron emission tomography,

single photon emission computed tomography, and functional magnetic resonance imaging have demonstrated

alterations in either cerebral perfusion or glucose metabolism in frontal, temporal, and basal ganglia areas of the

brain.23,24,25,26 Additional attention has been directed toward the correlation of regional brain abnormalities with

specific symptoms or subtypes of schizophrenia. Thus far, the most robust findings have been found in the relation

between negative symptoms and the prefrontal cortex, positive symptoms and temporal lobes, and thought disorder

and the planum temporale.27

In summary, neuroimaging studies have identified anatomical and functional abnormalities that affect different

areas of the brain, particularly the prefrontal and temporal areas, corresponding with the impairments observed in

schizophrenia.28 Medial temporal structures are important for the processing of sensory information, and

abnormalities in this area may explain distortions in the interpretation of external reality that are characteristic of

schizophrenia. The prefrontal cortex is responsible for some of the complex and highly evolved human functions,

including the integration of information from other cortical areas. The prefrontal cortex is largely responsible for

the regulation of working memory, which involves maintaining information in temporary memory while that

information is used for executive functions. Hence, these abnormalities in prefrontal areas could explain the deficits

in working memory and attention that are often present in schizophrenia. New theories of schizophrenia that have

been proposed include the “cognitive dysmetria” theory (implicating the thalamus and its cortical and cerebellar connections), the “disconnection

model” (implicating temporolimbic and prefrontal disconnections), and the “asynchronous neural firing”

model.29,30,31

Neurochemistry The discovery of the antipsychotic properties of chlorpromazine led to a fundamental understanding of the

neurochemistry of schizophrenia. The finding that chlorpromazine and other antipsychotic drugs decrease dopamine

activity by blocking specific postsynaptic receptors served as the foundation for the dopaminergic hypothesis of

P.78p3

Page 2 of 62Ovid: Applied Therapeutics: The Clinical Use Of Drugs

17/8/2010mk:@MSITStore:C:\Users\askari\Desktop\E%20Book\Applied\Applied%20Therapeuti...

Page 3: Schizophrenia

schizophrenia. Nearly all drugs that decrease dopamine activity decrease the positive symptoms of schizophrenia.

Furthermore, the clinical efficacy of antipsychotic agents is roughly proportional to their affinity for a particular

subtype of dopamine receptors, the D2 receptor.32,33 Although other classes of dopamine receptors have been

identified (e.g., D1, D3, D4), this close relationship to clinical potency exists only for the D2 receptor subtype.34

Indirect evidence supporting the dopamine hypothesis is found in the observation that dopamine agonists such as

amphetamine, levodopa, methylphenidate, and other aminergic agents can worsen schizophrenia symptoms in some

patients.35,36,37 Although these observations indicate that manipulating the dopaminergic system can regulate the

positive symptoms of schizophrenia, they do not directly implicate a central excess of dopamine as the sole cause of

schizophrenia symptoms. This simplistic theory does not explain, for instance, cases of schizophrenia in which

residual or negative symptoms predominate. The “hypofrontality theory” suggests that reduced or dysfunctional

dopaminergic neurotransmission within the prefrontal cortex or mesocortical area of schizophrenic patients may be

responsible for negative symptoms.38,39 In addition to negative symptomatology, prefrontal dopaminergic dysfunction

has been correlated with cognitive dysfunction, particularly in the area of working memory. It has also been

proposed that a chronically low level of striatal dopamine release causes negative symptoms (caused by low levels of

dopamine in the prefrontal cortex). The diminished dopamine release leads to an upregulation of dopamine

receptors, resulting in supersensitivity to phasic dopamine release in the context of environmental stressors

(exhibited behaviorally as positive symptoms).40 This theory explains why dopamine-blocking drugs improve positive

symptoms, but not negative symptoms.

Serotonin also seems to play a role in schizophrenia. Alteration in monoamine activity in the limbic circuit has been

proposed as a possible link to the dopamine hypothesis of schizophrenia.41 Serotonin receptors are abundant in

mesocortical areas, and agonism at these receptor sites may have an inhibitory effect on dopaminergic receptors or

dopamine release, possibly contributing to negative schizophrenia symptoms.

Another area of focus is the involvement of excitatory amino acids and the glutamate transmitter system in the

pathogenesis of schizophrenia.42 Decreased activation of N-methyl-D-aspartic acid (NMDA) or glutamate receptors

may increase cortical dopamine release and produce a syndrome resembling schizophrenia. This hypofunctional

NMDA receptor theory was supported by findings of low glutamate levels in the cerebrospinal fluid of persons with

schizophrenia and by the observation that phencyclidine (PCP), an NMDA antagonist, could produce florid

psychosis.43 In utero exposure to excitotoxins or viruses has been proposed as a possible mechanism for destruction

of NMDA receptors within the brain. The clinical effect of neuronal destruction may not be expressed until late

adolescence or early adulthood (when symptoms of schizophrenia usually arise). γ-Aminobutyric acid (GABA), a

major inhibitory amino acid, may act as a third link between the neuromodulation of glutamate and dopamine

receptors. It has been suggested that GABA receptor function may be impaired within the prefrontal region,

resulting in abnormal NMDA receptor concentrations.44 It has also been proposed that an imbalance between NMDA,

GABA, and dopamine receptors may cause the symptoms of schizophrenia. Studies examining the clinical effects of

direct glycinergic agonists or inhibitors of glutamate uptake in persons with schizophrenia are needed to clarify the

role of excitatory amino acid neurotransmission in this disorder.

Substantial progress has been made in neuropathological and neuroanatomical studies of schizophrenia. Insights from

physiological imaging studies in living patients, studies of brain development and its genetic control, and

pharmacologic studies using newer atypical antipsychotics are promising stepping stones that may lead to a better

understanding of the pathogenesis of schizophrenia.

Clinical Presentation

Historical Concept of Schizophrenia Kraepelin45 provided the first thorough description of the constellation of symptoms that make up schizophrenia,

which he called “dementia praecox” (a syndrome of cognitive and behavioral deficits that tend to appear early in

life). Kraepelin emphasized that dementia praecox generally followed a chronic course with no return to the

premorbid level of functioning. Kraepelin also noted that no single pathognomonic symptom or cluster of symptoms

served to characterize dementia praecox, an illness he considered so mysterious that he referred to it as a disorder

whose causes were shrouded in “impenetrable darkness.” Bleuler46 concurred with Kraepelin's initial description of

schizophrenia, although he suggested that some patients did recover and subsequently could lead productive lives.

For Bleuler, the most important and fundamental feature was a fragmentation in the formulation and expression of

thought, referring to it as “loosening of associations.” Bleuler described the “4 As” of schizophrenia, which include

autism (preoccupation with internal stimuli), inappropriate affect (external manifestations of mood), loose

associations (illogical or fragmented thought processes), and ambivalence (simultaneous, contradictory thinking).

Thus, Bleuler focused on negative symptoms and thought disorganization rather than on the positive symptoms of

Page 3 of 62Ovid: Applied Therapeutics: The Clinical Use Of Drugs

17/8/2010mk:@MSITStore:C:\Users\askari\Desktop\E%20Book\Applied\Applied%20Therapeuti...

Page 4: Schizophrenia

schizophrenia, such as delusions and hallucinations. Another important contribution to the defining features of

schizophrenia was Schneider's “first-rank” symptoms47 of schizophrenia that assisted the clinician in diagnosing

schizophrenia. First-rank symptoms include delusions, auditory hallucinations, thought withdrawal, thought

insertion, thought broadcasting, and experiencing feelings or actions that are under someone else's control.

Schneider's first-rank symptoms provided the framework for the systematic diagnostic criteria used today in

psychiatry.

Diagnosis and Differential Diagnosis The Diagnostic and Statistical Manual of the American Psychiatric Association, text revision (DSM-IV-TR), is the

latest edition of the guide for diagnosing and classifying schizophrenia and other psychiatric disorders. Compared

with previous DSM editions, the DSM-IV-TR places a greater emphasis on negative symptoms and the social and

occupational dysfunction associated with schizophrenia.11 Psychosis has many causes, and all must be excluded

before the diagnosis of schizophrenia is made. In addition, a diagnosis can be made only if the DSM-IV-TR criteria are

met (Table 78-1). Clinicians can misdiagnose psychotic patients when the diagnosis is based on presenting symptoms

alone without regard to the longitudinal clinical course. An accurate diagnosis is important because treatments vary

for psychoses with different origins. Schizophreniform disorder is similar to schizophrenia except that it lasts for >1

month but for <6 months. Brief psychotic disorder is diagnosed when positive symptoms present suddenly and are

present for ≥1 day but <1 month. After the episode subsides, premorbid functioning usually returns. Various

personality disorders, including schizotypal, schizoid, and paranoid types also can resemble schizophrenia; however,

these disorders lack the chronic thought disturbances seen in schizophrenia. Bipolar disorder, manic or depressive

phase, and major depression can have psychotic features that usually are mood congruent. Negative symptoms of

schizophrenia such as akinesia, anergy, apathy, and social withdrawal may resemble depression, but do not respond

to antidepressant therapy. It is also important to differentiate extrapyramidal side effects of drugs, particularly

akinesia seen with antipsychotic-induced parkinsonism, from the negative symptoms of schizophrenia. Common

differential diagnoses are listed in Table 78-2.

P.78p4

Table 78-1 DSM-IV-TR Criteria for Schizophrenia

A. Characteristic Symptoms

At least two of the following, each present for a significant portion of time during a 1-month period (or

less if successfully treated):

1. Delusions

2. Hallucinations

3. Disorganized speech (e.g., frequent derailment or incoherence)

4. Grossly disorganized or catatonic behavior

5. Negative symptoms (i.e., affective flattening, alogia, or avolition)

Note: Only one “A symptom” is required if delusions are bizarre or hallucinations consist of a voice

keeping up a running commentary on the person's behavior or thought, two or more conversations with

each other.

B. Social/Occupational Dysfunction

For a significant portion of the time since the onset of the disturbance, one or more major areas of

functioning such as work, interpersonal relationships, or self-care is markedly below the level achieved

before the onset.

C. Duration

Continuous signs of the disturbance persist for at least 6 months. This 6-month period must include at

Page 4 of 62Ovid: Applied Therapeutics: The Clinical Use Of Drugs

17/8/2010mk:@MSITStore:C:\Users\askari\Desktop\E%20Book\Applied\Applied%20Therapeuti...

Page 5: Schizophrenia

Drug-induced psychosis is an important differential diagnosis when evaluating patients with schizophrenia-like

symptoms. Illicit drugs may cause psychotic symptoms in any individual, but do not cause the illness of schizophrenia

in persons without a predisposition to mental illness or an underlying psychiatric disorder. Drugs causing acute

psychotic symptoms include amphetamines, cocaine, cannabis, phencyclidine (“angel dust”), lysergic acid

diethylamide, and ketamine. In addition, anticholinergic delirium can occur if excessive doses or combinations of

therapeutic agents with anticholinergic properties are prescribed. A urine toxicology screen can help to evaluate the

contribution of substance abuse to psychosis, but because of the risk of false-negative results, the quality and

progression of the presenting symptoms and physical findings also must be considered to make an accurate

diagnosis. A particularly difficult problem is evaluating the cause of psychosis in a patient with schizophrenia and

concurrent drug or alcohol abuse. Distinguishing between the two may not be possible until the patient becomes drug free and residual symptoms

are assessed. There are some differences in the presentation of drug-induced psychosis and schizophrenia. For

example, chronic stimulant use usually does not present with a formal thought disorder. Cannabis psychosis may be

differentiated from acute paranoid schizophrenia because patients with the former usually experience a subjective

feeling of panic and retain insight into their problem.48 Acute PCP ingestion can cause psychotic symptoms such as

paranoia and acute catatonia, but other concomitant symptoms such as ataxia, hyperreflexia, nystagmus, and

hypertension are not found in schizophrenia.

least 1 month of symptoms (or less if successfully treated) that meet criterion A (i.e., active-phase

symptoms) and may include prodromal and/or residual periods when the “A criterion” is not fully met.

During these periods, signs of the disturbance may be manifested by negative symptoms or by two or

more symptoms listed in “criterion A” present in an attenuated form (e.g., blunted affect, unusual

perceptual disturbances).

D. Schizoaffective Disorder and Mood Disorder Exclusion

Schizoaffective disorder and mood disorder with psychotic features have been ruled out because either

(a) no major depressive, manic, or mixed manic episodes have occurred concurrently with the active

phase symptoms or (b) if mood episodes have occurred during active phase symptoms, their total

duration has been brief relative to the duration of the active and residual periods.

E. Substance/General Medical Condition Exclusion

The disturbance is not due to direct physiological effects of a substance (drug of abuse or medication)

or a general medical condition.

F. Relationship to a Pervasive Development Disorder

If there is a history of autistic disorder or another pervasive development disorder, the additional

diagnosis of schizophrenia is made only if prominent delusions or hallucinations also are present for at

least a month (or less if successfully treated).

From reference 11.

P.78p5

Table 78-2 Differential Diagnosis for Schizophrenia (DSM-IV-TR)

Page 5 of 62Ovid: Applied Therapeutics: The Clinical Use Of Drugs

17/8/2010mk:@MSITStore:C:\Users\askari\Desktop\E%20Book\Applied\Applied%20Therapeuti...

Page 6: Schizophrenia

Target Symptoms Schizophrenia is a complex disorder comprising different clusters of signs and symptoms. The characteristic

symptoms of schizophrenia have often been conceptualized as falling into two broad categories: positive and

negative (or deficit) symptoms. Positive symptoms can be further divided into two distinct groups, positive

symptoms and disorganized symptoms. Positive symptoms include hallucinations (auditory, visual) and delusions

(persecution, guilt, religion, mind control). Disorganized symptoms include disorganized speech, thought disorder

(tangentiality, derailment, circumstantiality) and disorganized behavior (clothing, appearance, aggression,

repetitive actions). Negative symptoms include affective flattening, decreased thought and speech productivity

(alogia), loss of ability to experience pleasure (anhedonia), and decreased initiation of goal-directed behavior

(avolition). Most patients with schizophrenia exhibit both positive and negative symptoms, although the dominance

Drug-induced Psychoses

Amphetamine

Cocaine

Cannabis (marijuana)

Phencyclidine (PCP)

Lysergic acid diethylamide (LSD)

Anticholinergics

Primary Psychiatric Disorders

Brief psychotic disorder

Schizophreniform disorder

Bipolar affective disorder, manic type

Mood disorder with psychotic features

Personality Disorders

Schizotypal

Schizoid

Paranoid

Page 6 of 62Ovid: Applied Therapeutics: The Clinical Use Of Drugs

17/8/2010mk:@MSITStore:C:\Users\askari\Desktop\E%20Book\Applied\Applied%20Therapeuti...

Page 7: Schizophrenia

of one type over the other usually varies throughout the course of the illness. Younger patients tend to exhibit more

positive symptoms, whereas in older patients negative symptoms predominate. Table 78-3 provides a glossary of

commonly used terms in schizophrenia, and Table 78-4 lists positive and negative symptoms of schizophrenia.

Table 78-3 Glossary of Commonly Used Terms in Schizophrenia

Affect: behavior (usually an expression of an emotion) observed by the interviewer. Common types of

disturbances in affect include: restricted—mild decrease in range and intensity of the expression of

emotion; blunted—significant decrease in intensity of the expression of emotion; flat—absence of

expression of emotion; inappropriate—incongruency between patient's affect and mood or behavior; and

labile—abrupt shifts in expression of emotion.

Akathisia: syndrome consisting of subjective feelings of anxiety and restlessness, and objective signs of

pacing, rocking, and an inability to sit or stand still for extended periods of time.

Akinesia: absence or decrease in voluntary movement; may be antipsychotic induced (extrapyramidal

side effects) or a manifestation of negative symptoms of schizophrenia.

Alogia: impoverished thinking usually manifested through speech and language deficits. Speech is brief

and lacks spontaneity; replies to questions are very concrete (poverty of speech). Poverty of content

refers to speech that is adequate in amount, but is of little substance (overly abstract), repetitive, or

stereotyped.

Anergy: lack of energy.

Anhedonia: loss of interest or pleasure.

Avolition: an inability to initiate and sustain goal-directed activities. The patient may sit for extended

periods of time and show minimal interest in participating in social or work-related activities.

Circumstantiality: a form of disorganized speech characterized by “talking in circles” or taking an

unusually long length of time in answering a question or expressing one's point of view.

Delusions: a false belief that is firmly held despite evidence to refute the belief. The belief does not

qualify as a delusion if it is a cultural or religious belief accepted by a group of individuals. Types of

delusions include grandiose, persecutory, and somatic type.

Executive function: the ability to design and carry out a solution to a plan when the solution is not

obvious. Loss of executive function presents as failure to learn from past experience and failure to plan

or organize life events.

Hallucination: a sensory perception (e.g., auditory, visual, somatic, tactile) experienced in the absence

of external stimuli. Hallucinations may be recognized as false sensory perceptions in some, whereas

others may believe that the experiences are reality based.

Loose associations: a form of disorganized, illogical speech characterized by unrelated words, phrases,

Page 7 of 62Ovid: Applied Therapeutics: The Clinical Use Of Drugs

17/8/2010mk:@MSITStore:C:\Users\askari\Desktop\E%20Book\Applied\Applied%20Therapeuti...

Page 8: Schizophrenia

Cognitive deficits, including problems with attention, memory, and concentration, are another frequently cited

problem for people with schizophrenia. These impairments are common, affecting as many as 70% of individuals with

schizophrenia.49 The most consistently replicated deficits are observed on tasks measuring attention, information

processing speed, working memory, learning, and executive (frontal systems) functions.50 These domains are not

only commonly impaired in schizophrenia, but they are also strongly associated with functional outcomes (social and

community functioning).50 Although it is likely that the overall dysfunction observed in schizophrenia encompasses

factors such as psychopathology, drug treatment, and social isolation, evidence suggests that cognitive impairments

have a stronger relationship to the long-term functional outcome than positive symptoms.50

and sentences used in a fashion that makes comprehension very difficult, if not impossible.

Mood: a pervasive and sustained emotion that is experienced by the patient. Examples include

depressed, anxious, angry, or irritable mood.

Mood-congruent delusions or hallucinations: delusions or hallucinations that are consistent with a

mood or behavior (e.g., delusions/hallucinations of death, guilt, or punishment in the presence of a

depressed mood).

Mood-incongruent delusions or hallucinations: delusions or hallucinations that are not consistent with

a mood or behavior (e.g., delusions/hallucinations of death, guilt, or punishment in the presence of

mania).

Tangentiality: a form of disorganized speech in which answers are remotely or completely unrelated to

questions, and patients' thoughts frequently shift in an unconnected fashion.

Thought broadcasting: a delusion that one's thoughts are being broadcast to others (e.g., a patient

feels that others can read his or her mind).

Thought disorder: a general term often used to describe any type of abnormal thought process (e.g.,

delusion, loose association, conceptual disorganization).

Thought insertion: a delusion that one's thoughts are being inserted into one's mind by others.

P.78p6

Table 78-4 Positive and Negative Symptoms of Schizophrenia

Positive Negative

Combativeness, agitation, and hostility Psychomotor retardation

Affective flattening

Tension Avolition

Hyperactivity Lack of socialization

Page 8 of 62Ovid: Applied Therapeutics: The Clinical Use Of Drugs

17/8/2010mk:@MSITStore:C:\Users\askari\Desktop\E%20Book\Applied\Applied%20Therapeuti...

Page 9: Schizophrenia

Case Presentation

J.C., a 22-year-old, single, unemployed man, was brought into the emergency department (ED) of a mental

health hospital by the police after he was found running barefoot downtown dodging cars in subzero weather.

In the ED, J.C. was extremely frightened, and stated, “They won't let me go.”

History of Present Illness: J.C. is not a poor historian. He is agitated easily and threatens the interviewer when

asked questions regarding his illness. J.C. said he came to this city 5 years ago “to be the king of jazz music.”

He did not complete high school, has lost touch with his family and has been living at the Salvation Army for the

past 3 years. He has no close friends, and does not trust most people. He hears voices of “dead people” who

tell him he is worthless and “will be killed.” He says, “The newscasters on the television are reading my mind

and telling everyone my personal secrets.” These problems and insomnia have been disturbing him for the past

year.

Medical History: J.C. has no history of a previous psychiatric illness and has never been hospitalized. The

administrator at the Salvation Army confirms that J.C. has been living at the facility for 3 years. His behavior

has always been rather “odd and unusual”; he has recently become more agitated. He does not get along with

other tenants and staff and he has been unable to care for himself for the past year or so.

Psychosocial History: The hospital social worker reveals that J.C.'s mother was frequently hospitalized for

unspecified psychotic episodes and his father was “never around.” J.C. admits that he was somewhat of a

“rebel” as a child; he has never held steady employment and relies on Social Security income and money from

strangers to survive.

Physical Examination/Laboratory Tests: J.C. was given a complete physical examination, which was

noncontributory. His laboratory tests, including complete blood count (CBC) with differential, SMA-28

laboratory panel, and thyroid and liver function tests (LFTs) were within normal limits. The urine drug screen

was negative, his neurologic examination was unremarkable, and no further tests were ordered.

Mental Status Examination: Appearance and Behavior: J.C. is a thin, disheveled-looking man with very poor

hygiene, who appears much older than his stated age. He is extremely suspicious of the interviewer and his

surroundings and continually asks, “Where am I, who are you, and what do you want?” He also is agitated

easily. Mood: J.C. is very anxious and worried, concerned that the “dead people” are going to track him down

and “bury me alive.” His affect is blunted, with a minimal range of reactivity to his emotions. Memory: Remote

memory seems to be intact, although his immediate and short-term memory are difficult to assess because he is

uncooperative. Sensorium: He is oriented to person, place, time, and situation. Thought Content: He hears

dead people talking to him. He seems to be responding to internal stimuli, mumbling to himself and answering

his own questions. Thought Process: J.C. is very suspicious and exhibits loose associations such as “What are

you doing to me? Don't you like my hat? Jazz musicians will save us all. You are with the FBI!” Insight and

Judgment: Both are poor as evidenced by J.C.'s denial of his illness and need for treatment, and his behavior,

which precipitated this admission.

Provisional Diagnosis: Schizophrenia, Paranoid Type

1. What target symptoms of schizophrenia are present in J.C.? How are target symptoms used to monitor

Hallucinations Alogia

Delusions Loss of emotional connectedness

Disorganized speech (loose associations, tangential, blocking) Loss of executive functions

Unusual behavior �

Page 9 of 62Ovid: Applied Therapeutics: The Clinical Use Of Drugs

17/8/2010mk:@MSITStore:C:\Users\askari\Desktop\E%20Book\Applied\Applied%20Therapeuti...

Page 10: Schizophrenia

treatment?

J.C. exhibits many of the classic Schneider's first-rank symptoms of schizophrenia, including auditory hallucinations

(hearing voices from “dead people” telling him that he will be killed), delusions (“Jazz musicians will save us all.

You are with the FBI. They will bury me alive.”), and thought broadcasting (“The newscasters on the television are

reading my mind.”). Other target symptoms include agitation, suspiciousness, loose associations, poor grooming and

hygiene, impaired sleep, decreased social skills, and impaired insight and judgment. These target symptoms are the

basis of assessing a change in clinical status and monitoring J.C.'s response to medications.

Typical Course and Outcome

Onset of Illness

The onset of schizophrenia starts with the prodromal phase that begins with the first changes in behavior and lasts

until the onset of psychosis.51,52,53 It is characterized by a slow and gradual onset of signs and symptoms that can last

weeks to years, but typically persists for at least a year.51 Common prodromal signs and symptoms include anxiety,

blunted affect, depression or dysphoric mood, irritability, loss of initiative, low energy, poor concentration, sleep

disturbance, social isolation and withdrawal, and suspiciousness. Subthreshold or attenuated positive symptoms, and

mild disorganization (digressive, vague, overly abstract, or concrete thinking) may begin during the prodrome.

Gradual deterioration in functioning commonly occurs. The prodrome is difficult to recognize; however, family

members may interpret the earliest subtle changes as normal adolescence, a phase, relational problems, depression,

or drug use. Some researchers have viewed the prodromal phase as the optimal point to begin pharmacologic

intervention. Research has shown that many patients are ill for months or even years before seeking treatment; this

delay in treatment is related to outcome and level of remission. Early recognition of the prodrome could lead to

interventions that delay or prevent the emergence of psychosis. Eventually, a symptom characteristic of the active

phase appears (Table 78-1, criterion A), marking the disturbance as schizophrenia.

Course of Illness

In contrast with the notion of an inevitable progressive deterioration in schizophrenia as proposed by Kraepelin, longitudinal studies of schizophrenia suggest that the course of illness is highly variable. The majority suffers from

episodic exacerbations and remissions with some degree of residual symptoms.45,54,55,56 Complete remission (i.e., a

return to full premorbid functioning) is not common in this disorder. Of the patients who remain ill, some seem to

have a relatively stable course, whereas others show a progressive worsening associated with severe disability.

Reported remission rates range from 10% to 60%, and a reasonable estimate is that 20% to 30% of all schizophrenic

patients are able to lead somewhat normal lives. About 20% to 30% of patients continue to experience moderate

symptoms, and 40% to 60% of patients remain significantly impaired by their disorder for their entire lives. Early in

the illness, negative symptoms may manifest as prodromal features. Subsequently, positive symptoms appear.

Because these positive symptoms are particularly responsive to treatment, they typically diminish, but in many

individuals negative symptoms persist between episodes of positive symptoms. As patients with schizophrenia age,

the frequency and severity of acute episodes may decrease; however, negative symptoms may become more

prominent in some individuals, a state often referred to as burn-out.54,56,57

Prognosis

Predictors of an improved long-term outcome in patients with schizophrenia include female gender, a positive family

history of an affective disorder, a negative family history of schizophrenia, good premorbid functioning, higher IQ,

married marital status, acute onset with precipitating stress, fewer prior episodes (both number and length), a

phasic pattern of episodes and remissions (i.e., fewer residual symptoms), advancing age at onset, minimal

comorbidity, paranoid subtype, and symptoms that are predominantly positive (delusions, hallucinations) and not

disorganized (thought disorder, disorganized behavior).58 Despite modern treatment advancements, schizophrenia

remains a severe disease with a relatively poor outcome. Many patients experience repeated hospitalizations and

exacerbations of symptoms, and suicide attempts are relatively common. Individuals with schizophrenia have a 20%

shorter life expectancy than the general population.59 This is due in part, to an increased incidence of general

medical illness (such as diabetes or cardiovascular disease) and suicide.58 Patients with schizophrenia have

approximately a 50% lifetime risk for a suicide attempt and as many as 9% to 13% complete suicide.60

2. What other factors in J.C.'s history are consistent with the diagnosis of schizophrenia, and what is his

prognosis?

Other factors that are consistent with schizophrenia in J.C. include onset of illness in early adulthood (typical age of

P.78p7

Page 10 of 62Ovid: Applied Therapeutics: The Clinical Use Of Drugs

17/8/2010mk:@MSITStore:C:\Users\askari\Desktop\E%20Book\Applied\Applied%20Therapeuti...

Page 11: Schizophrenia

onset is late adolescence to early 30s), a positive family history of psychiatric illness, inability to maintain steady

employment and establish interpersonal relationships with others, and the apparent chronicity of the illness over the

past year. His acute exacerbations of psychosis are also characteristic of schizophrenia. Because of J.C.'s young age

at the time of his initial psychotic episodes, a positive family history for thought disorder, an unstable home

environment, and a low premorbid level of functioning, his long-term prognosis is not good.

Treatment Schizophrenia is a chronic disease for which there is no cure. Pharmacotherapy can reduce symptoms to improve

social and cognitive functions; however, patients have multiple relapses and experience residual symptoms

throughout their lives. Treatment can decrease acute symptoms, decrease the frequency and severity of psychotic

episodes, and optimize psychosocial functioning between episodes. However, many patients require comprehensive

care and life-long treatment. Comprehensive care consists of pharmacologic, psychosocial, and rehabilitative

treatment.

Goals of Treatment The American Psychiatric Association Practice Guideline for the Treatment of Schizophrenia describes three phases

of illness for the purpose of integrating treatment.58 Treatment is divided into three phases—acute, stabilization,

and stable—with each phase using different drug and nondrug strategies to achieve the desired therapeutic outcome.

These phases are not always distinct or separate, but instead tend to overlap.

Acute Phase

During the acute phase of illness, patients suffer from floridly psychotic symptoms such as delusions and/or

hallucinations, are severely disorganized, and usually require hospitalization or are placed in a supervised outpatient

setting. Negative symptoms become more severe during this phase as well. The initial treatment goal is to calm

agitated patients who may be physical threats to themselves or others. Medication is usually required to achieve this

outcome, although nondrug interventions such as emotional support from the staff and use of quiet areas can also be

helpful. An antipsychotic should be initiated as soon as feasible because prolonged psychotic episodes may be

associated with a worsening of their course of illness.61

Stabilization Phase

During the stabilization phase, acute symptoms gradually decrease in severity as the patient begins to stabilize. The

goals of treatment during the stabilization phase of the illness are to reduce the likelihood of symptom exacerbation

and develop a plan for long-term treatment.58 The hope is that a patient will return to his or her premorbid level of

functioning, although this is unlikely. Treatment should consist of a combination of pharmacologic and

nonpharmacologic strategies. Treatment guidelines recommend that patients continue the same medication and

dosage from which they received benefit from during the acute phase of illness. Treatment should be continued for

at least 6 months.58 During this phase, the individual is most vulnerable to relapse. Another concern during the

stabilization phase is that clinicians, patients, and families may wrongly conclude that the antipsychotic stopped

working if psychotic symptoms persist during this phase. For example, a patient may have suffered from

hallucinations that were loud and derogatory before initiation of treatment; after 8 weeks of treatment, the voices

might remain but become quieter and less demeaning. Because antipsychotics have been shown to provide gradual

improvement over a period of months after the initial episode was treated, the current regimen should be

continued.62

Stable Phase

During the stable phase, a level of functioning considered optimal for that individual patient is attained. Although

complete resolution of symptoms is desired, many patients never achieve this goal. Negative symptoms may persist

and patients may experience nonspecific symptoms such as tension, anxiety, or mood instability during the stable

phase. Treatment during the stable or maintenance phase is designed to optimize functioning and minimize the risk

and consequences of relapse. Antipsychotics are highly effective in preventing relapse. There are no reliable

predictors of relapse for an individual patient. When developing a long-term strategy to prevent relapse and

rehospitalization, the dose and type of antipsychotic, and the duration of therapy should be considered. Long-term

pharmacotherapy reduces the risk of relapse in schizophrenic patients and also may reduce the risk of environment-

and stress-induced relapse relative to untreated patients.63,64 The effectiveness of antipsychotics in remission

prevention was demonstrated in studies in which stable patients were either continued on an antipsychotic or

changed to placebo. In these studies, relapse rates were higher in patients who did not remain on an antipsychotic.

P.78p8

Page 11 of 62Ovid: Applied Therapeutics: The Clinical Use Of Drugs

17/8/2010mk:@MSITStore:C:\Users\askari\Desktop\E%20Book\Applied\Applied%20Therapeuti...

Page 12: Schizophrenia

Although results vary, approximately 70% of the patients who were switched to placebo experienced a relapse within

12 months. In contrast, only about 30% of drug-maintained patients had a relapse of their condition.65 This supports

the need for continuing antipsychotic treatment in patients after they have recovered from a psychotic episode. All

patients with schizophrenia should receive maintenance therapy for at least 1 year, unless antipsychotic agents are

not tolerated or the diagnosis is uncertain. If a patient has only a single episode with predominately positive

symptoms and is symptom free for 1 year after the acute episode, then discontinuation of the medication with

careful follow-up can be considered.58 Patients with a history of multiple episodes may be candidates for medication

discontinuation after stability has been demonstrated for 5 years.58 Lifelong treatment is indicated in patients who

present a significant risk to themselves or others when unmedicated.58

3. What are the specific treatment goals for J.C.?

The immediate treatment goal for J.C. is to reduce his agitation because he may be a physical threat to himself and

others. Intermediate goals during the stabilization phase are to attenuate, or to eliminate, if possible, symptoms of

psychoses and the thought disorder. Long-term goals during the stable phase should include assisting J.C. in

developing a psychosocial support system (e.g., caretakers, mental health workers, peer groups) to promote

enhanced medication adherence, enable him to live semi-independently, and possibly obtain part-time employment.

Nonpharmacologic Interventions Nonpharmacologic interventions are often combined with drug treatment and can provide additional benefits in such

areas as relapse prevention, improved coping skills, better social and vocational functioning, and ability to function

more independently. Interventions should be started as early as possible, even during the management of an acute

episode. As a patient begins to stabilize during an acute episode, nonpharmacologic strategies can be

implemented.58 Individual therapy (e.g., supportive, insight oriented, reality oriented) can improve insight into the

illness, improve medication adherence, teach ways to cope with medication side effects and stress, and help the

patient to identify early warning signs of relapse. Group therapy can enhance socialization skills. In patients who are

less stable and who continue to exhibit negative symptoms, supportive therapy is generally more effective than

group or other more complex, insight-oriented therapies. Family therapy is also important because family members

need to learn ways to cope with such a devastating illness and how to be supportive of their loved one, while not

being overly controlling. Vocational training can benefit patients who will likely need a significant amount of

assistance in finding and maintaining long-term employment.66 Evidence-based practice guidelines for psychosocial

treatment of schizophrenia now include interventions such as social skills training, cognitive/cognitive–behavioral

therapy, family psychoeducation, and vocational rehabilitation.67,68,69,70,71

4. Is nonpharmacologic therapy indicated for treatment of J.C.?

J.C. should be placed in an area of the hospital ward with low amounts of noise and stimulation until his agitation

subsides. He needs careful observation by staff until he is fully assessed for the cause of his symptoms and his level

of dangerousness. If the treatment team determines that he is very dangerous, he may need one-to-one supervision

by an individual staff member. As his behavior calms, J.C. can receive more ward privileges and should begin to

attend group therapy. Initial therapy focuses on issues such as “what is schizophrenia?” and “what are the ways to

treat it?” Basic issues related to medication education are also discussed. These include symptoms that are improved

with medication and recognition of common acute adverse effects. Socialization with other patients and staff should

also be encouraged. As J.C. improves further, therapy and education can shift toward coping skills, stress

management, recognition of prodromal symptoms of relapse, long-term adverse effects, and ways to enhance

adherence to treatment. Family therapy should also be considered if J.C. allows contact to occur. Family therapy

focuses on decreasing family stress surrounding the illness, family problem solving, and communication. If J.C.

enters a stable phase after discharge from the hospital, other strategies such as psychosocial clubhouses to improve

socialization, social skills training, and vocational rehabilitation can increase his quality of life and productivity.58

5. How should the staff and family members communicate most effectively with J.C., considering his paranoid

state?

Communication should be calm and nonjudgmental in any acutely psychotic and agitated patient. Open-ended

questions should be used, with a switch to closed-ended questions if J.C. is unwilling to respond. His paranoid

delusions should not be challenged directly because he may feel attacked and become angry and more agitated.

Instead of challenging the delusional thoughts, the clinicians should acknowledge that J.C. believes the “dead

people” are real and that he is “the king of jazz,” but that they do not share his belief. Direct eye contact should

also be intermittent to avoid making the patient feel threatened. If the patient is violent, they should be

interviewed in the presence of another health care provider. The clinician must not react to the provocation of a

threatening patient and should avoid using a loud voice or aggressive words. When interviewing

Page 12 of 62Ovid: Applied Therapeutics: The Clinical Use Of Drugs

17/8/2010mk:@MSITStore:C:\Users\askari\Desktop\E%20Book\Applied\Applied%20Therapeuti...

Page 13: Schizophrenia

a violent patient, questions should be focused on issues that need immediate attention, such as medication

adherence before admission to the hospital, presence of medical conditions, and recent consumption of drugs and

alcohol.72,73 As J.C. calms, he will likely be more cooperative with questions and assessments.

Assessment Because a diagnosis of schizophrenia cannot be made or monitored by laboratory or physical tests, the clinician must

use target symptoms gathered from a patient interview and historic records to assess treatment response. A

complete medical, psychiatric, and medication history, physical examination, laboratory panel (electrolytes;

glucose; liver, renal, and thyroid function tests; CBC with differential; urinalysis; urine toxicology screen), and

electrocardiogram (ECG) should be obtained as soon as possible during the acute phase of illness or once the patient

is cooperative (e.g., stable phase). A complete evaluation is needed to rule out other causes of psychosis, to identify

comorbid conditions requiring treatment, and to give guidance for selecting an antipsychotic and for determining

the length of treatment.

Given the lack of insight that characterizes schizophrenia patients, collateral information should be gathered from

the patient's relatives, friends, and significant others. Target symptoms for schizophrenia are specific for an

individual patient, and they must be clearly identified and documented before and during the course of treatment to

determine response to an intervention. Both specific treatment goals and the patient's previous baseline level of

functioning should be clearly established at the onset of drug therapy. These data are used to give a direction for

treatment and as a way of determining whether the outcome has been achieved.

Pharmacologic Interventions

Non-Antipsychotic Agents

Benzodiazepines

Benzodiazepines are commonly added to antipsychotics and have been found to be useful in some studies for

anxiety, agitation, global impairment, and psychosis.74 Adjunctive use of benzodiazepines can spare the need for

higher dosages of antipsychotics. However, some studies found that the benefits of benzodiazepines were sometimes

not sustained.58 Unfortunately, it is not possible to predict who will respond to adjunctive benzodiazepine

treatment, and any potential benefit must be balanced against the risks of benzodiazepines. Common side effects of

benzodiazepines include sedation, ataxia, cognitive impairment, and behavioral disinhibition. This latter effect can

be a serious problem in those patients who are being treated for agitation. Withdrawal reactions, including psychosis

and seizures, can significantly complicate management if a patient suddenly becomes noncompliant with

benzodiazepine treatment. In addition, patients with schizophrenia are vulnerable to both abuse and addiction to

benzodiazepines. For these reasons, the use of benzodiazepines should be limited to short trials (2–4 weeks in

duration) for the management of severe agitation and anxiety.

Lithium

The use of lithium in schizophrenia has been investigated as monotherapy as well as an adjunctive treatment with

antipsychotics. Studies evaluating the antipsychotic properties of lithium monotherapy indicate that it has limited

effectiveness in schizophrenia and may be harmful for some patients.58 In combination with antipsychotics, lithium

has been observed to improve psychosis, depression, excitement, and irritability.58,75,76 In general, adjunctive

lithium therapy may be considered in patients who have a partial or poor response to an antipsychotic agent. The

dose of lithium should be sufficient to obtain a blood level in the range of 0.8 to 1.2 mEq/L. Patients should be

monitored for adverse effects that are commonly associated with lithium (e.g., polyuria, tremor). Reports of

increased neurotoxicity from combined use of lithium and antipsychotic agents are inconclusive, and the risk seems

to be no different with either medication used alone.77

Anticonvulsants

Anticonvulsants such as carbamazepine and valproate are often prescribed in patients with schizophrenia as

adjunctive treatment of psychosis, agitation, aggression, impulsivity, and mood lability.78,79 However, as with

lithium, there is little support for the efficacy of these agents as monotherapy. In general, the evidence to support

the use of carbamazepine for schizophrenia is inconsistent. Neppe80 reviewed the literature and concluded that

patients with nonresponsive psychosis and agitation, aggression, or “interpersonal difficulties” may benefit from the

addition of carbamazepine. In a recent meta-analysis, Leucht et al. described a nonsignificant trend for a benefit

P.78p9

Page 13 of 62Ovid: Applied Therapeutics: The Clinical Use Of Drugs

17/8/2010mk:@MSITStore:C:\Users\askari\Desktop\E%20Book\Applied\Applied%20Therapeuti...

Page 14: Schizophrenia

from carbamazepine as an adjunct to antipsychotics.78 An important consideration with carbamazepine is that the

drug alters the metabolism of most antipsychotic agents, and dosage adjustment is often required. Carbamazepine

should never be used concurrently with clozapine because of the additive risk of agranulocytosis.

Recent evidence indicates that valproate is being prescribed more frequently for schizophrenia and that the use of

lithium and carbamazepine is declining.81 This shift in utilization may reflect a more favorable side effect profile of

valproate. In a recent investigation of hospitalized patients treated with risperidone and olanzapine for an acute

exacerbation of schizophrenia, concurrent administration of divalproex resulted in earlier improvements in a range

of psychotic symptoms.82 Additional research is needed to confirm this finding. Dosages and target serum levels of

carbamazepine and valproate are similar to those useful in the treatment of bipolar disorder (see Chapter 80 Mood

Disorders II: Bipolar Disorders).

Propranolol

Propranolol, in combination with antipsychotic drugs, has been evaluated in nonresponders, with variable results.

Potential explanations for the enhanced efficacy include a drug interaction leading to higher antipsychotic drug

serum concentrations, relief of akathisia, or a primary improvement in psychosis. In addition, propranolol has

improved chronic aggression in individuals with schizophrenia.83 When used to treat aggression or to enhance the

response to antipsychotic agents, high doses are needed. Propranolol is started at 40 to 80 mg twice a day and is

increased every other day until intolerable adverse effects occur (systolic blood pressure [BP] of <90 mm Hg or a

heart rate <50 beats/minute). Dosages as low as 160 mg and as high as 1,920 mg have been effective; however, a

reasonable trial usually is considered to be 240 mg/day for 2 months.83,84

Antipsychotics

Classification and Nomenclature of the Antipsychotics

Antipsychotics have been broadly classified into two groups. The older agents (i.e., those introduced in the United

States before 1990) are referred to as typical or conventional antipsychotics or dopamine receptor antagonists, with

pharmacologic activity attributed to the blockade of central dopamine receptors, particularly the D2 receptor

subtype. These agents have also been referred to as major tranquilizers and neuroleptics. The term major

tranquilizer is inaccurate, because these agents, particularly the high-potency agents, can improve psychosis

without associated sedation. Neuroleptic refers to the tendency of these drugs to cause neurologic side effects,

particularly extrapyramidal symptoms (EPS). Typical antipsychotics are further classified as high- or low-potency

agents, based on their relative ability to block dopamine receptors. They can also be classed by chemical structure

(phenothiazine and nonphenothiazine) and potential for common adverse effects (EPS, sedation, anticholinergic, and

cardiovascular effects).66,85 Examples of typical agents include haloperidol, fluphenazine, thiothixene,

chlorpromazine and thioridazine.

Newer agents, atypical or serotonin-dopamine antagonists, consist of clozapine, risperidone, olanzapine,

quetiapine, ziprasidone, aripiprazole, and paliperidone. These agents demonstrate postsynaptic effects at 5-HT2A

and D2 receptors. Some of the general characteristics that may be considered as features of atypical antipsychotic

include an absence or decreased incidence of EPS and tardive dyskinesia (TD), lack of effect on serum prolactin,

greater efficacy for refractory schizophrenia, and greater activity against negative symptoms.86 However, currently

there are no widely accepted characteristics that define an “atypical” antipsychotic, and clozapine is actually the

only atypical agent which fulfills all of the criteria stated above. Recently, the term second-generation

antipsychotic has also been proposed to describe this class of medications, although there is no consensus on the

nomenclature at the present time.87

Mechanism of Drug Action

Although the specific mechanism of action of antipsychotics has not been elucidated, the focus is postsynaptic

blockade at dopamine D2 and serotonin 5-HT2A receptor sites. It is generally accepted that D2 receptor antagonism

plays a key role in the treatment of positive symptoms of schizophrenia as well as in the production of EPS and

hyperprolactinemia-related side effects. Knowledge of the central dopamine pathways in the brain can be used as a

model to understand the therapeutic and side effects of the antipsychotics. The central dopamine system is

composed of four tracts: mesolimbic, mesocortical, nigrostriatal, and tuberoinfundibular (Fig. 78-1). Drug action can

be predicted if a clinician understands the function of each tract, along with the binding affinity of an agent for

receptors located in the tract (Tables 78-5 and 78-6). Blockade of dopamine receptors in the mesolimbic tract is

likely responsible for the reduction of positive symptoms of schizophrenia. Blockade of the other dopamine tracts is

largely responsible for the adverse effects of antipsychotic treatment. The mesocortical tract is responsible for

P.78p10

Page 14 of 62Ovid: Applied Therapeutics: The Clinical Use Of Drugs

17/8/2010mk:@MSITStore:C:\Users\askari\Desktop\E%20Book\Applied\Applied%20Therapeuti...

Page 15: Schizophrenia

higher-order thinking and executive functions. Dopamine hypofunctioning in this area, either from the schizophrenia

itself or by antipsychotic action, may be responsible for negative symptoms. The nigrostriatal tract modulates body

movement. Antipsychotic-induced blockade in this area causes EPS. Lastly, antipsychotic-induced blockade of the

dopamine tract in the tuberoinfundibular area of the anterior pituitary leads to hyperprolactinemia.39 Ideally,

selective dopaminergic blockade of the mesolimbic tract would be preferred. Unfortunately, typical antipsychotics

block all four of these dopaminergic pathways. Studies have demonstrated that antipsychotic effects require a

striatal D2 receptor occupancy of 65% to 70%; D2 receptor occupancy greater than 80% significantly increases the risk

of EPS.88,89 During chronic treatment with typical antipsychotic agents, between 70% and 90% of D2 receptors in the

striatum are usually occupied.90 Not surprisingly, all the typical antipsychotic agents have been observed to be

effective in reducing positive symptoms of schizophrenia and they all can cause EPS and TD.

In contrast with typical antipsychotic agents that affect all four dopamine tracts, atypical antipsychotics primarily

affect dopamine tracts in the limbic system and have been termed limbic specific.39 For example, therapeutic doses

of risperidone, olanzapine, and ziprasidone produce >70% occupancy at D2 receptors.91,92 However, attributing

antipsychotic efficacy solely to D2 receptor effects is an oversimplification because many patients do not respond to

medication despite adequate D2 occupancy.88 Furthermore, low levels of D2 striatal receptor occupancy (<70%) have

been observed with therapeutic doses of clozapine and quetiapine. This may explain the low propensity of these

agents to produce EPS, but also calls into question the minimum receptor occupancy necessary for antipsychotic

efficacy as proposed for typical agents.92,93

Blockade of the 5-HT2A receptors, a shared property of the atypical agents, has been investigated with regard to its

significance in mediating antipsychotic effects.94 At therapeutic doses, most atypical agents occupy more than 80%

of cortical 5-HT2A receptors.92,95,96 Blockade of 5-HT2A receptors, independent of D2 antagonism, has not been

demonstrated to produce antipsychotic effects. However, it is postulated that a high 5-HT2A to D2 receptor affinity

ratio may underlie the enhanced therapeutic efficacy and low propensity for EPS observed with atypical

antipsychotics.39 Serotonin is known to exert a regulatory effect on dopaminergic receptors or dopamine release, but

the degree of control may vary depending on the pathway. Specifically, serotonin tonically inhibits dopamine

release. Therefore, 5-HT2A antagonism should enhance dopaminergic transmission. It has been proposed that the

atypical properties of antipsychotics (efficacy against negative symptoms and low propensity to produce EPS) are

due in part to augmentation of dopaminergic function via 5-HT2A blockade in the mesocortical and nigrostriatal

pathways and limbic specificity.39,97 Aripiprazole is an antipsychotic agent with unique effects at D2 and 5-HT

receptors. Specifically, this agent has partial agonist activity at D2 and 5-HT1A receptors and antagonist activity at

serotonin 5-HT2A receptors. Aripiprazole is a functional antagonist at D2 receptors under hyperdopaminergic

conditions but exhibits functional agonist properties under hypodopaminergic conditions.98 Paliperidone (Invega) is

the newest atypical agent approved by the U.S. Food and Drug Administration (FDA). Because paliperidone is the

primary active metabolite of risperidone, 9-hydroxyrisperidone, we should expect the mechanism of action of

paliperidone to be similar to that of risperidone.99,100

P.78p11

Page 15 of 62Ovid: Applied Therapeutics: The Clinical Use Of Drugs

17/8/2010mk:@MSITStore:C:\Users\askari\Desktop\E%20Book\Applied\Applied%20Therapeuti...

Page 16: Schizophrenia

Figure 78-1 Four dopamine pathways in the brain. The neuroanatomy of dopamine neuronal pathways in

the brain can explain both the therapeutic effects and the side effects of the known antipsychotic agents. (1)

The nigrostriatal dopamine pathway projects from the substantia nigra to the basal ganglia, and is thought to

control movements. (2) The mesolimbic dopamine pathway projects from the midbrain ventral tegmental

area to the nucleus accumbens, a part of the limbic system of the brain thought to be involved in many

behaviors, such as pleasurable sensations, the powerful euphoria of drugs of abuse, as well as delusions and

hallucinations of psychosis. (3) A pathway related to the mesolimbic dopamine pathway is the mesocortical

dopamine pathway. It also projects from the midbrain ventral tegmental area, but sends its axons to the

limbic cortex, where it may have a role in mediating positive and negative psychotic symptoms or cognitive

side effects of neuroleptic antipsychotic medications. (4) The fourth dopamine pathway of interest is the one

that controls prolactin secretion, called the tuberoinfundibular dopamine pathway. It projects from the

hypothalamus to the anterior pituitary gland.

Table 78-5 Neurotransmitter Tract Function and Effect of Typical Versus Atypical Antipsychotics

Neurotransmitter Tract Clinical Function Typical Agent Effects Atypical Agent Effects

Dopamine-

nigrostriatal

Modulates EPS Potent D2

blockade

causes EPS

Minimal EPS owing to greater

specificity for mesolimbic

system

Dopamine-

mesolimbic

Modulates arousal,

memory, behavior

Effectively

treats positive

symptoms

Effectively treats positive

symptoms

Page 16 of 62Ovid: Applied Therapeutics: The Clinical Use Of Drugs

17/8/2010mk:@MSITStore:C:\Users\askari\Desktop\E%20Book\Applied\Applied%20Therapeuti...

Page 17: Schizophrenia

Dopamine-

mesocortical

Modulates cognition,

socialization, and

other negative

symptoms

Less effective

for negative

symptoms

Clozapine (possibly others)

greater efficacy for negative

symptoms; clozapine,

risperidone (others?) may

improve cognition

Dopamine-

tuberoinfundibular

Regulates prolactin

release

Increases

prolactin

release (dose

related)

No increase in prolactin with

clozapine, dose-related with

risperidone, no significant

increase with others

Serotonergic (5-

HT2)

5-HT2 blockade

reduces EPS,

improves negative

symptoms

Minimal

affinity for 5-

HT2 receptors

Greater affinity for 5-HT2

receptors

EPS, extrapyramidal side effects.

Adapted from references 39, 86, and 97.

P.78p12

Table 78-6 Relative Receptor-Binding Affinities of Typical and Atypical Antipsychotic Agents

Receptor D1

D2 5-HT2 α

1M

1H

1

Typical Agents

Chlorpromazine - +++ ++ +++ +++ ++

Fluphenazine - +++ + + - -

Perphenazine - +++ ++ ++ - ++

Thioridazine + +++ ++ +++ +++ +

Haloperidol ++ +++ + + - -

Atypical Agents

Clozapine ++ ++ +++ +++ +++ +

Risperidone - +++ +++ +++ - +

Page 17 of 62Ovid: Applied Therapeutics: The Clinical Use Of Drugs

17/8/2010mk:@MSITStore:C:\Users\askari\Desktop\E%20Book\Applied\Applied%20Therapeuti...

Page 18: Schizophrenia

Antipsychotic drugs also affect other receptors, usually resulting in adverse effects rather than enhanced

therapeutic response. For example, histamine-1 (H1) blockade is responsible for sedation and possibly weight gain.

Blockade of 5-HT2C receptors may also play a role in the weight gain associated with some atypical agents. α1-

Adrenergic blockade causes orthostatic hypotension and plays a role in sexual dysfunction. Muscarinic receptor

blockade causes the classical anticholinergic effects such as dry mouth, blurred vision, constipation, and urinary

retention. An understanding of the receptor-binding properties of the typical and atypical agents is helpful to

predict their adverse effects profiles.66,85,101 Properties of typical and atypical antipsychotics are listed in Tables 78-

6 and 78-7.

Olanzapine ++ ++ +++ ++ +++ ++

Quetiapine - + ++ +++ + +

Ziprasidone ± ++ +++ ++ - +

Aripiprazole + +++ ++ ++ - +

Paliperidone - +++ +++ +++ - +

D1, dopamine subtype 1; D2, dopamine subtype 2; 5-HT2, serotonin subtype 2; 1, alpha 1; M1,

muscarinic (cholinergic) subtype 1; H1, histamine subtype 1.

-, none; ±, unclear; +, minimal; ++, moderate; +++, high; ++++, very high.

Compiled from references 90, 99, 100, 143, and 266.

Table 78-7 Relative Incidence of Antipsychotic Drug Adverse Effects

� Sedation EPS Anticholinergic Orthostasis Seizures

Prolactin

Elevation

Weight

Gain

Typical—Low Potency

Chlorpromazine ++++ +++ +++ ++++ +++ +++ ++

Thioridazine ++++ ++ ++++ ++++ ++ +++ +++

Typical—High Potency

Trifluoperazine ++ ++++ ++ ++ +++ +++ ++

Fluphenazine ++ +++++ ++ ++ ++ +++ ++

Thiothixene ++ ++++ ++ ++ ++ +++ ++

Page 18 of 62Ovid: Applied Therapeutics: The Clinical Use Of Drugs

17/8/2010mk:@MSITStore:C:\Users\askari\Desktop\E%20Book\Applied\Applied%20Therapeuti...

Page 19: Schizophrenia

Selection of an Antipsychotic In all patients, the selection of a specific antipsychotic agent should be individualized. The choice of a specific

medication is usually based on a number of factors and is frequently more art than science. Important factors

include prior experience (efficacy and side effects), the ease of attaining a therapeutic dose, available dosage

forms, and formulary or cost considerations. One of the best predictors of response is the patient's previous response

to a specific antipsychotic, or a positive response in a first-degree relative. If a patient responded poorly to an

antipsychotic in the past, an agent from a different chemical class should be prescribed. Also, if a patient has a

particular aversion to an antipsychotic side effect, an appreciation for differences among the available

antipsychotics in this regard would be helpful. Drugs with a similar side effect profile should be avoided. Although

all the antipsychotic agents have been shown to be efficacious during the acute phase of schizophrenia, atypical

antipsychotics (with the exception of clozapine) have become the agents of first choice for the treatment of

schizophrenia. The rationale for this practice is based on the lower risk of EPS and TD with the atypical agents.

Efficacy

Haloperidol + +++++ + + ++ +++ ++

Loxapine +++ ++++ ++ +++ ++ +++ +

Molindone + ++++ ++ ++ ++ +++ +

Atypicals

Clozapine ++++ + ++++ ++++ ++++c 0 ++++

Risperidone +++ +a ++ +++ ++ 0 to

+++c

++

Olanzapine +++ +b +++ ++ ++ +c +++

Quetiapine +++ + ++ ++ ++ 0 ++

Ziprasidone ++ + ++ ++ ++ 0 +

Aripiprazole ++ + ++ ++ ++ 0 +

Paliperidone +++ +a ++ +++ ++ 0 to

+++c

++

aVery low at dosages <8 mg/day.

bWith dosages <20 mg/day.

cDose related.

0, no effect; +, very low; ++, low; +++, moderate; ++++, high; +++++, very high; EPS, extrapyramidal

side effects.

Compiled from references 58, 99, 100, 182, 143, 166, and 266.

P.78p13

Page 19 of 62Ovid: Applied Therapeutics: The Clinical Use Of Drugs

17/8/2010mk:@MSITStore:C:\Users\askari\Desktop\E%20Book\Applied\Applied%20Therapeuti...

Page 20: Schizophrenia

Typical antipsychotics are effective in reducing positive symptoms of schizophrenia during acute psychotic episodes

and in preventing recurrence in many patients.102 These agents are less effective for treating negative symptoms. In

fact, there is concern that typical agents may exacerbate negative symptoms by causing drug-induced akinesia. In

general, all typical antipsychotic agents are believed to be equally effective when used in equivalent doses.

Clinical trials demonstrate that the atypical agents—clozapine, risperidone, olanzapine, quetiapine, ziprasidone,

aripiprazole, and paliperidone—are superior to placebo, and are at least as effective as typical antipsychotics for

treatment of the positive and negative symptoms of schizophrenia.98,103,104,105,106,107,108,109,110,111,112,113 In a meta-

analysis of 52 randomized clinical trials comparing atypical agents (including clozapine, risperidone, quetiapine)

with typical agents (haloperidol or chlorpromazine), Geddes et al.114 found no difference in efficacy between these

two drug classes if the dose of the typical agent was considered. The apparent superiority of atypical antipsychotics

(in terms of efficacy and drop-out rates) could be attributed to the use of excessive doses of typical agents (>12

mg/day haloperidol or equivalent). In contrast, two other meta-analyses concluded that the atypical agents have

efficacy and tolerability advantages over typical agents independent of haloperidol dosage.115,116 Davis et al.116

concluded that available evidence demonstrates an efficacy advantage for some (clozapine, risperidone, olanzapine)

but not all atypicals (quetiapine, ziprasidone, and aripiprazole) when compared with typical agents. These authors

suggest that the atypical agents are at least as efficacious as typical agents for positive symptoms, and that atypical

agents are likely more efficacious with negative and cognitive symptoms. This potential advantage of the atypical

agents continues to be a source of much debate. On one hand, there are some who believe that atypical agents have

a unique therapeutic effect on the primary negative symptoms, whereas others contend that this finding is actually a

secondary impact upon other related symptoms and disease manifestations.117 Data on risperidone and olanzapine

suggest a direct effect on primary negative symptoms independent of their effect on psychotic, depressive, or

EPS.118,119

The National Institute of Mental Health Clinical Antipsychotic Trials of Intervention Effectiveness trial (CATIE) was a

real-world trial evaluating schizophrenic patients. The trial compared the effectiveness of atypical antipsychotics

(quetiapine, risperidone, clozapine, olanzapine, and, later, ziprasidone) to that of perphenazine, a midpotency

typical antipsychotic.120 Perphenazine was chosen owing to a lower incidence of EPS compared with high and other

midpotency typical antipsychotics, as well as decreased sedation when compared with low-potency typical

antipsychotics. In this study, 1,460 patients were included from 57 sites in 24 states. The results indicated

olanzapine was least likely to be discontinued; however, it (and clozapine) had the most metabolic side effects.

Perphenazine was as effective as the atypical agents, however, it had the highest incidence of EPS, even after

patients with TD were excluded from receiving the medication.121,122

Dosage Forms

Oral Disintegrating Tablet

Pharmacotherapy during the acute phase is administered orally (as tablets or liquid concentrate) or intramuscularly

(IM), depending on the patient's willingness to take medication, the risk of imminent harm, and dosage form

availability. Usually, antipsychotic therapy is administered orally. The newest technology available is the oral

disintegrating tablet (ODT). ODT can be in the form of a compressed tablet, compression molded tablets, and

freeze-dried (lyophilized) wafers that dissolve or disintegrate within a matter of seconds when placed on the tongue.

Of the atypical agents, risperidone, olanzapine, and aripiprazole are available as ODT.

Short-Acting Injectables

Although all of the antipsychotics are available in oral formulations, only a few are available as injectables. Short-

acting IM preparations of an antipsychotic may be preferred over oral medication if a patient is agitated and not

likely to be cooperative. IM preparations bypass the gastrointestinal tract and first-pass metabolism and have a

faster onset of action. For example, most IM antipsychotics reach a maximum plasma level within 30 to 60 minutes

and patients usually experience substantial calming within 15 minutes.123 Of the short-acting agents available for IM

use, most clinical experience has been with high-potency typical antipsychotic medications such as haloperidol or

fluphenazine. Haloperidol and fluphenazine have the advantage of being calming without being sedating, but they

can cause severe EPS. Some clinicians prefer low-potency agents such as chlorpromazine, because they are more

sedating. Low-potency agents can also cause severe hypotension. High-potency agents can be given in larger doses

because they have a lower risk of anticholinergic and cardiovascular complications than low-potency agents. In

general, the use of oral typical agents has declined because of the neurologic risks. Studies have shown that the

FDA-approved IM formulations of the atypical agents (olanzapine, aripiprazole, and ziprasidone) seem to be better

tolerated than typical agents and are equally effective as haloperidol in the acute treatment of psychoses in

Page 20 of 62Ovid: Applied Therapeutics: The Clinical Use Of Drugs

17/8/2010mk:@MSITStore:C:\Users\askari\Desktop\E%20Book\Applied\Applied%20Therapeuti...

Page 21: Schizophrenia

patients with schizophrenia.124,125

Agitation is a problem that many mental health providers face. It can affect whether or not a patient can be

treated at any point in time. A patient who is acutely agitated may behave in a way that may result in harm of self,

others, or property. In these cases, medication is needed to calm the patient. In the past, sleep was thought to be

essential in the treatment of acute agitation. This is no longer true, though, some facilities may still use this as an

endpoint or goal.126 If rapid tranquilization is required for a patient who is acutely agitated and exhibiting dangerous

behavior, the combination of IM haloperidol and IM lorazepam appears more effective than either alone.127 However,

rapid tranquilization is not necessarily more effective than traditional dosing methods, and is associated with a

greater incidence of acute side effects such as dystonic reactions.128,129 Rapid tranquilization can be achieved when

repeated IM injections of high-potency antipsychotics or benzodiazepines are given until the patient is either

adequately calmed, a maximum recommended daily dose is reached, or dose-limiting adverse effects such as acute

dystonia (antipsychotics), ataxia, or slurred speech (benzodiazepines) occur. The most commonly used regimen is

haloperidol 2 to 5 mg combined with lorazepam 2 mg injected IM every 30 to 60 minutes to a maximum of three

doses.130 Patients should be monitored for severe adverse events, particularly those patients with underlying medical

or neurologic problems. Vital signs should be monitored and the dose should be held if clinically significant adverse

effects occur.129,131 The concept of rapid tranquilization should not be confused with rapid neuroleptization, which

implies the use of very high loading dosages of antipsychotics (e.g., administering a series of closely spaced IM doses

over a period of hours) to produce a more rapid remission of psychotic symptoms.130 The practice of rapid

neuroleptization leads to a higher incidence of EPS without advantages of efficacy when compared with the

administration of lower doses of the same drugs. Rapid neuroleptization is no longer recommended.128 Most recently,

with the advent of IM and fast-acting oral formulations of atypical antipsychotics, the combination has changed to

decrease the incidence of EPS. Clinical trials have shown that risperidone, ziprasidone, aripiprazole, and olanzapine

are at least as effective as haloperidol and lorazepam (Table 78-8).125,126,132,133,134,135,136,137,138,139

P.78p14

Table 78-8 Agents to Treat Acute Agitation

Medication Dosage Form Dose Onset Half-life Duration of Action

Lorazepam PO (tablet), IM, IV 1–2 mg 60–90 min 12–15 hr 8–10 hr

Typical Antipsychotics

Haloperidol PO (tablet), IM, IV 5–10 mg 30–60 min 12–36 hr up to 24 hr

Droperidol IM, IV 5–10 mg 15–30 min 2–4 hr 6–8 hr

Atypical Antipsychotics

Olanzapine PO (tablet), IM, ODT 10 mg 15–45 min 30 hr 24 hr

Risperidone PO (tablet, liquid), ODT 2 mg 60 min 20 hr Not available

Ziprasidone PO (tablet), IM 20 mg 30–60 min 2–5 hr 4 hr

Aripiprazole PO (tablet, liquid), IM 9.75 mg 1–3 hr 75–94 hr Not available

IM, intramuscularly; IV, intravenously; ODT, oral disintegrating tablet; PO, orally.

Page 21 of 62Ovid: Applied Therapeutics: The Clinical Use Of Drugs

17/8/2010mk:@MSITStore:C:\Users\askari\Desktop\E%20Book\Applied\Applied%20Therapeuti...

Page 22: Schizophrenia

6. How should pharmacotherapy be used to treat the acute phase of schizophrenia in J.C., and what should be

done first to manage his acute symptoms?

Aripiprazole 9.75 mg IM injection can be given to J.C. so further assessment can be completed. Aripiprazole for any

a typical IM agent is a good choice because it has efficacy in calming rather than sedating the patient, it has a lower

propensity for EPS and TD, and is available in a short-acting IM formulation. These are important features because

J.C. is likely to refuse oral medications because of his paranoid symptoms. Also, aripiprazole IM has limited

cardiovascular problems such as hypotension, bradycardia with or without hypotension, tachycardia, and syncope as

compared with olanzapine IM.132,139 Lorazepam 0.5 to 1 mg PO or IM every 4 to 6 hours as needed may be given to

J.C. to control ongoing agitation, however, greater sedation and orthostatic hypotension have been observed with

the combination of lorazepam and aripiprazole as compared to aripiprazole alone.132 Subsequent doses of

aripiprazole IM can be given every 2 hours if needed for agitation, up to a maximum of 30 mg/day.

7. J.C. was given a single dose of aripiprazole 9.75 mg IM. His agitation resolved after the first injection but you

are later informed that this medication is not covered by J.C.'s healthplan. What is the next medication you

would recommended for J.C., and what factors should be considered when selecting an antipsychotic for the

acute and stabilization phases?

An antipsychotic prescribed at a dose reflective of the needs during the stabilization phase should be started now.

Olanzapine, 10 mg at bedtime, is an appropriate choice for pharmacologic treatment of schizophrenia in the acute

phase. With the exception of clozapine, any of the other atypical agents would also be appropriate choices.

Past experience with antipsychotics predicts therapeutic success in the selection of a regimen. J.C. does not have a

past psychotropic medication history to guide drug selection. He also does not have a history of nonadherence,

which would direct the choice toward a depot product. A common starting dose for olanzapine is 10 mg/day.

8. When should J.C.'s target symptoms start to respond to olanzapine?

During the first week, often known as the medicated cooperation stage, J.C. should respond to the calming

properties of the antipsychotic, and his symptoms of hostility, agitation, and insomnia should improve. During the

next 2 to 6 weeks, the improved socialization stage, J.C. should begin to obey hospital rules, attend ward meetings,

and generally become more sociable. Severely ill schizophrenic patients with chronic disease may never reach this

stage. The elimination of the thought disorder stage can occur within any time frame, but usually takes at least 2 to

3 weeks and up to several months to occur. During this stage, the core symptoms of schizophrenia such as delusions,

hallucinations, and thought disturbance improve. If no improvement of J.C.'s core symptoms is observed after 3

weeks of olanzapine 10 mg/day, then the dose should be increased slowly to 15 or 20 mg/day and the patient should

be observed for another 3 to 4 weeks.

9. It is 6 months later. Most of J.C.'s symptoms have improved, he is residing in assisted living housing and

working part-time at Goodwill Industries. Although he is not troubled by any side effects, he sees no point in

continuing his medication. Should olanzapine be discontinued? What are the current practice guidelines

regarding long-term treatment?

J.C. clearly is responding well to olanzapine and has no side effects. To minimize risk of relapse, he should continue

his current regimen for at least another 6 months. If his symptoms remain stable and he maintains good psychosocial

Compiled from references 125, 126, and 132, 133, 134, 135, 136, 137, 138, 139.

P.78p15

Page 22 of 62Ovid: Applied Therapeutics: The Clinical Use Of Drugs

17/8/2010mk:@MSITStore:C:\Users\askari\Desktop\E%20Book\Applied\Applied%20Therapeuti...

Page 23: Schizophrenia

functioning (e.g., continues work, keeps close contact with social workers, attends medication groups),

discontinuation of therapy can be considered.

When J.C. has been stabilized for a total of approximately 1 year on olanzapine 10 mg/day, then the dose can be

reduced by 20% every 3 to 6 months until it is discontinued or relapse occurs. If he exhibits recurrent target

symptoms of schizophrenia or decompensation, then the dose should be increased to the previous effective dose (or

medication restarted if it was discontinued). Because it is likely that J.C. will require lifelong antipsychotics, it is

important to determine the minimal effective dose to prevent recurrence, while minimizing the risk of adverse

effects. J.C. is not a candidate for intermittent therapy during the stable phase. He is not a decanoate candidate

because he is compliant with treatment.

Long-Acting Depot Injectables

Long-acting depot medications are not recommended for acute psychotic episodes because these medications take

months to reach a steady-state concentration and are eliminated very slowly.58 Hence, it is difficult to correlate

clinical effect with dosage, and it is extremely difficult to make dosage adjustments to manage side effects.

However, long-acting depot medications can be useful for maintenance therapy in patients with a history of

nonadherence to their oral medication and in those who prefer the convenience of long-acting or depot injections.

In these situations, the oral form of available depot medications should be initiated first. If the oral form has been

shown to be safe and effective, the patient can be converted to the depot form. Fluphenazine, haloperidol, and

risperidone are available as long-acting decanoate injections. Long-acting injectable risperidone appears to combine

the most valuable features of an atypical antipsychotic (broadly efficacious and well tolerated) with those of

injectable long-acting antipsychotics (improved bioavailability and assured medication delivery).141,142

Pharmacokinetics and Drug Interactions

Dosing considerations also have a major influence on safe and effective antipsychotic selections. Specific factors

include the number of times a day a medication needs to be administered, the difference between a starting dose

and a “target” therapeutic dose (i.e., titration required), and the risk for drug–drug interactions. The long half-life

(12–24 hours) and active metabolites of most oral antipsychotic drugs allow for once- to twice-daily dosing.143 Most

antipsychotics, with the exception of clozapine and ziprasidone, can be safely given once a day. Most antipsychotics

achieve a steady-state concentration in 4 to 7 days, but it is important to understand that the onset of antipsychotic

effect is not related to achieving steady state. Pharmacologic effects often persist for longer than pharmacokinetics

would imply. In some instances, treatment is initiated at a subtherapeutic dose and gradually titrated upward to an

effective dose. This approach is taken to allow the patient to develop tolerance to adverse events such as sedation

or orthostatic hypotension. In the case of acute agitation, antipsychotic doses are also often divided, despite the

long half-lives of these drugs, such that the sedative effects are maintained over the day. Once a patient has been

stabilized or has become tolerant to the adverse effects, the goal is to give medication once a day, usually at

bedtime. Bedtime dosing enhances medication adherence and concentrates ongoing adverse effects such as sedation

at night.

Both pharmacokinetic and pharmacodynamic drug interactions can occur with antipsychotic agents. Cytochrome

P450 enzymes, especially the 1A2, 2D6, and 3A4 isoenzymes, are responsible for the metabolism of many

antipsychotics.144 Induction or inhibition of these enzymes by other drugs may result in clinically important drug

interactions. Table 78-9 summarizes the metabolic pathways for commonly used antipsychotic drugs. In the case of

clozapine, serious complications such as seizures can occur when drug interactions cause serum concentrations of

clozapine to rise significantly. Examples of drugs that have the potential to cause serious interactions with

antipsychotics include fluvoxamine (CYP 1A2 inhibitor), erythromycin, ketoconazole, and ritonavir (CYP 3A4

inhibitors), quinidine, risperidone, fluoxetine, and paroxetine (CYP 2D6 inhibitors), and cimetidine (multiple enzyme

inhibition).145 Conversely, inducers of drug metabolism such as carbamazepine (induces multiple enzymes) or

cigarette smoking (induces CYP 1A2) can reduce the plasma concentrations of antipsychotic drugs. For example,

carbamazepine has been shown to reduce plasma concentrations of haloperidol by 50%.146 Cigarette smoking has

been shown to increase the metabolism of clozapine and olanzapine.144 Pharmacodynamic interactions may occur

when medications with similar unwanted properties are concurrently prescribed. For example, concomitant use of

low-potency typical agents or clozapine along with diphenhydramine or hydroxyzine may cause augmentation of

anticholinergic effects. Pharmacodynamic drug interactions of greatest concern are those that can cause significant orthostatic hypotensive, anticholinergic effects, and sedation. Pharmacokinetic and dosing information

for the antipsychotics are provided in Tables 78-9 and 78-10.

P.78p16

Page 23 of 62Ovid: Applied Therapeutics: The Clinical Use Of Drugs

17/8/2010mk:@MSITStore:C:\Users\askari\Desktop\E%20Book\Applied\Applied%20Therapeuti...

Page 24: Schizophrenia

Table 78-9 Pharmacokinetic Comparisons of Antipsychotics

Antipsychotic Agent

Mean Half-Life

(hr)

Major Cytochrome P450

Pathway Plasma Concentration Range

Chlorpromazine 8–35 2D6 Not well defined

Thioridazine 9–30 2D6 Not well defined

Perphenazine 8–21 2D6 Not well defined

Fluphenazine 14–24 2D6 0.2–2.8 mg/mL

Fluphenazine

decanoate

8 days 2D6 0.2–2.8 mg/mL

Thiothixene 34 2D6 2–15 mg/mL

Haloperidol 12–36 2D6 4–12 ng/mL

Haloperidol decanoate 21 days 2D6 4–12 ng/mL

Loxapine 5–15 None Not well defined

Molindone 10–20 None Not well defined

Clozapine 16 1A2, 3A4 350–420 µg/mL suggested

Risperidone 22 2D6 Not well defined

Olanzapine 30 1A2 >23.2 ng/mL @ 12 hours post

dose

Quetiapine 7 3A4 Not well defined

Ziprasidone 4–5 3A4 Not well defined

Aripiprazole 75–94 2D6, 3A4 Not well defined

Paliperidone 23 Limited 2D6, 3A4 Not well defined

Compiled from references 99, 132, 143, 144, 248, and 266.

Page 24 of 62Ovid: Applied Therapeutics: The Clinical Use Of Drugs

17/8/2010mk:@MSITStore:C:\Users\askari\Desktop\E%20Book\Applied\Applied%20Therapeuti...

Page 25: Schizophrenia

Pharmacoeconomic Considerations

Although medication expenditures represent only a small portion of total resource utilization associated with the

management of schizophrenia, concern exists about the cost of atypical antipsychotics and whether their relative

advantages over typical agents are worth their increased cost.147 Drug acquisition costs for atypical antipsychotics

can be 100-fold higher than typical antipsychotics. Most pharmacoeconomic studies with atypical antipsychotics

show that these agents are at least cost neutral and may offer cost advantages compared with traditional agents

when total mental health costs are considered.148 Depending on the individual study, the higher costs associated

with atypical antipsychotics are offset by decreased number of hospital admissions, length of inpatient stay, and

number of outpatient visits. Hence, drug costs should not be the sole factor considered when selecting an

antipsychotic. The selection of a medication should be individualized and factors such as efficacy, side effects,

patient acceptance, and total health care costs should be considered.

Adverse Effects

An important factor in the selection of an antipsychotic is the potential risk for adverse events. Key adverse effects

that differentiate the antipsychotics include EPS, anticholinergic side effects, cardiac effects, hyperprolactinemia,

metabolic effects and sedation. Table 78-7 compares antipsychotic agents with regard to the risk of these adverse

effects.

Extrapyramidal Side Effects and Tardive Syndromes

EPS is a broad term that describes several types of acute and chronic drug-induced movement disorders. Acute

dystonia, parkinsonism, and akathisia all occur early in treatment, whereas TD, tardive dystonia, and tardive

akathisia have a late onset, usually after years of treatment. The acute forms of EPS usually develop soon after the

initiation of antipsychotics, are dose dependent, and are generally reversible soon after discontinuation of the

offending agent.149 It has been estimated that 60% of patients who receive typical antipsychotics develop some form

of EPS acutely.58 In general, typical agents are more likely to cause EPS than atypical agents when these medications

are used at usual therapeutic doses. Among the seven currently available atypical agents, clozapine and quetiapine

are associated with the lowest risk for EPS. The rising prescription rate of atypical antipsychotics (and decrease in

use of the typical agents) have substantially reduced the problem of EPS. Similarly, the risk of antipsychotic-induced

TD is suggested to be lower with atypical agents compared with typical agents. Most of the evidence documenting a

decreased incidence of TD with atypical agents is derived from data and clinical experience with clozapine,

risperidone, olanzapine, and quetiapine.150,151,152,153

Acute Dystonia

Acute dystonia has the earliest onset of all the EPS symptoms. Most cases occur within the first few hours or days

after initiation or dose increase of antipsychotic medication. Dystonia is characterized by sustained muscle

contractions. Common presentations of antipsychotic-induced dystonia include a sudden onset of brief or sustained

abnormal postures, including tongue protrusion, oculogyric crisis (eyes rolling back into head), trismus (spasm of the

jaw), torticollis (torsion of the neck); opisthotonos (arching of back), and unusual positions in the trunk, limbs, and

toes. Laryngeal dystonias are the most serious dystonias and are potentially fatal. Risk factors for acute dystonia

include younger age, male gender, high dosage of high-potency typical antipsychotics, and previous history of

dystonia.58

The exact pathophysiology of acute dystonia is uncertain. Conflicting theories describe either a hypodopaminergic or

hyperdopaminergic state after an antipsychotic-induced blockade of postsynaptic dopamine receptors. Acute

dystonia likely is caused by dysregulation of the dopamine system and imbalance between neurotransmitter systems after acute antipsychotic administration.154 P.78p17

Table 78-10 Antipsychotic Relative Potency and Adult Dosing

Drug and Chemical Class

Dose

Equivalence

Usual Starting

Dose (mg/day)

Acute Phase

Dosage (mg/day)

Maintenance/Stable Phase

Dosage (mg/day)

Typical Agents–Phenothiazines

Page 25 of 62Ovid: Applied Therapeutics: The Clinical Use Of Drugs

17/8/2010mk:@MSITStore:C:\Users\askari\Desktop\E%20Book\Applied\Applied%20Therapeuti...

Page 26: Schizophrenia

Aliphatic type � � � �

���Chlorpromazine

(Thorazine)

100 50–200 300–1,500a 150–800

Piperidine type � � � �

���Thioridazine (Mellaril) 100 50–200 300–800 150–600

Piperazine type � � � �

���Perphenazine (Trilafon) 10 4–16 32–64a 8–48

���Trifluoperazine

(Stelazine)

5 2–10 10–80 5–30

���Fluphenazine (Prolixin) 2 2–10 5–80 2–20

Typical Agents–Nonphenothiazines

Thioxanthene � � � �

���Thiothixene (Navane) 4 2–10 5–60a 5–30

Butyrophenone � � � �

���Haloperidol (Haldol) 2 2–10 5–100 2–20

Dibenzoxazepine � � � �

���Loxapine (Loxitane) 10 10–20 50–250c 25–100

Dihydroindolone � � � �

���Molindone (Moban) 10 25–75 25–225 25–100

Diphenylbutylpiperidone � � � �

���Pimozide (Orap) 1 1–2 10–30 2–6

Page 26 of 62Ovid: Applied Therapeutics: The Clinical Use Of Drugs

17/8/2010mk:@MSITStore:C:\Users\askari\Desktop\E%20Book\Applied\Applied%20Therapeuti...

Page 27: Schizophrenia

10. J.P., a 19-year-old man, was brought to the psychiatric ED because of assaultive behavior toward his

mother. J.P. states he struck her because the devil told him to do it. Trifluoperazine 5 mg IM Q 4 to 6 hr PRN

was started to control his assaultive behavior. He received four IM injections over 24 hours and was converted

to 15 mg PO HS. On day 2 of the admission he complained of a stiff neck and protruding tongue. J.P. became

very upset and wanted to leave the hospital and never take these medications again. No other medical

conditions were noted. What evidence suggests that J.P. is experiencing an acute dystonia reaction?

The sudden appearance of a stiff neck and a protruding tongue in J.P. is consistent with acute dystonia. In addition,

J.P.'s young age, male gender, and use of a high-potency typical antipsychotic agent also place him at high risk for

experiencing a dystonic reaction.

11. How should acute dystonia be treated in J.P.?

Acute dystonic reactions are sudden in onset, dramatic in appearance, and can cause patients great distress. It

requires immediate treatment. The initial goal of treatment is to relieve symptoms as soon as possible with either

benztropine 1 to 2 mg or diphenhydramine 25 to 50 mg by IM injection. Although the IV route has a faster onset of

action, it is not needed in J.P. because his reaction is not severe. If symptoms do not resolve within 15 to 30

minutes, then the dose should be repeated. If there are contraindications to the use of anticholinergic drugs,

lorazepam 1 to 2 mg IM can be used. J.P. must be reassured that this is a temporary condition that can be prevented

and treated. To prevent another reaction, J.P. should be given oral anticholinergic drugs in doses commonly used for

pseudoparkinsonism for 2 weeks after this dystonic reaction.154

Parkinsonism

The clinical presentation of antipsychotic-induced parkinsonism includes bradykinesia or akinesia, which may be

associated with decreased arm swinging, a masklike face, drooling, decreased eye blinking, and soft, monotonous

speech, tremor, which is most commonly a rhythmic, resting tremor, and rigidity of the extremities, neck or trunk

(most identifiable in the limbs as a “cogwheel” rigidity during passive motion).154 Antipsychotic-induced

parkinsonism occurs in approximately 20% of patients treated with antipsychotic agents.58 Symptoms can occur at

any time, but usually develop within 4 weeks after antipsychotic initiation or a dose increase. Advanced age, antipsychotic dose and potency, and preexisting EPS are the major risk factors for antipsychotic-induced

Atypical Agents

Risperidone (Risperdal) 2 1–2 2–16 2–8

Clozapine (Clozaril) 50 12.5–25 150–900 150–600

Olanzapine (Zyprexa) 5 5–10 10–20 10–20

Quetiapine (Seroquel) 75 50–100 300–750 400–600

Ziprasidone (Geodon) 60 40–80 80–200 80–160

Aripiprazole (Abilify) 7.5 10–15 10–30 15–30

Paliperidone (Invega) 3 6 6–12 6–12

aDosages can be exceeded with caution, but high-dose therapy is rarely needed.

Compiled from references 57, 66, 80, 144, 151, 239, 241, and 256.

P.78p18

Page 27 of 62Ovid: Applied Therapeutics: The Clinical Use Of Drugs

17/8/2010mk:@MSITStore:C:\Users\askari\Desktop\E%20Book\Applied\Applied%20Therapeuti...

Page 28: Schizophrenia

parkinsonism.155 The Simpson Angus Rating Scale is used commonly to assess the presence and severity of EPS (Table

78-11).156

Antipsychotic-induced parkinsonism (as well as idiopathic Parkinson's disease) is thought to be due to postsynaptic

dopamine receptor blockade in the nigrostriatal system, leading to an imbalance between the dopaminergic and

cholinergic systems. Reasons for the delay in onset between receptor blockade, which occurs within hours after

initiating an antipsychotic, and development of symptoms are not well understood.154

Atypical antipsychotics are the mainstay of the pharmacologic management of psychosis in patients vulnerable to

developing antipsychotic-induced parkinsonism. The reduced parkinsonian liability of atypical antipsychotics has

been associated with their limbic specificity, 5-HT2A blocking effects and in some cases less D2 receptor

blockade.89,92,97

12. S.B., a 46-year-old man, has a diagnosis of chronic undifferentiated schizophrenia and was treated with

loxapine 50 mg TID with inadequate response. The dose cannot be increased due to oversedation. Because of

the incomplete response, S.B. was switched to haloperidol 5 mg Q AM and 10 mg QHS. One week after the

switch, S.B. returns to the clinic complaining of feeling “real slow.” He has a bilateral hand tremor that

improves when he picks up his coffee cup. Physical examination detected cogwheel rigidity in both arms,

although it was worse on the right side. S.B. wants to be taken off this “bad” medication. What evidence

suggests that S.B. has antipsychotic-induced parkinsonism?

The onset of symptoms within 1 week of starting a new, high-potency typical antipsychotic is the first clue to the

Table 78-11 Common Rating Instruments for Schizophrenia and Antipsychotics

Psychosis

Brief Psychiatric Rating Scale (BPRS)

Positive and Negative Symptom Scale for Schizophrenia (PANSS)

Scale for Assessment of Positive Symptoms (SAPS)

Scale for Assessment of Negative Symptoms (SANS)

Movement Disorders—Tardive Dyskinesia

Abnormal Involuntary Movement Scale (AIMS)

Dyskinesia Identification System Condensed User Scale (DISCUS)

Movement Disorders—Parkinsonism

Simpson Angus Scale for Extrapyramidal Symptoms

Movement Disorders—Akathisia

Barnes Akathisia Scale

Page 28 of 62Ovid: Applied Therapeutics: The Clinical Use Of Drugs

17/8/2010mk:@MSITStore:C:\Users\askari\Desktop\E%20Book\Applied\Applied%20Therapeuti...

Page 29: Schizophrenia

presence of EPS. The “slow feeling” possibly is indicative of akinesia, and S.B.'s bilateral tremor and cogwheel

rigidity also are features of antipsychotic-induced parkinsonism.

13. What antiparkinsonian drug could be selected for S.B., if any?

In mild cases of antipsychotic-induced parkinsonism, immediate intervention may not be required if the movement

disorder is not bothersome to the patient. For troublesome cases, as experienced by S.B., the simplest intervention

is to reduce the antipsychotic (haloperidol) dose to the lowest effective level. If dose reduction is not possible, then

an antiparkinsonian agent can be added (Table 78-12).154

All anticholinergic antiparkinsonian agents are equally effective for antipsychotic-induced parkinsonism, although

Table 78-12 Agents to Treat Antipsychotic-Induced Parkinsonism and Akathisia

Medication Equivalent Dose (mg) Dose/Day (mg)

Anticholinergic

Benztropine (Cogentin)a 0.5 2–8

Biperiden (Akineton)a 0.5 2–8

Diphenhydramine (Benadryl)a 25 50–250

Procyclidine (Kemadrin) 1.5 10–20

Trihexyphenidyl (Artane) 1 2–15

Dopaminergic

Amantadine – 100–300

GABAminergic

Diazepam (Valium) 10 5–40

Clonazepam (Klonopin) 2 1–3

Lorazepam (Ativan)a 2 1–3

Noradrenergic Blockers

Propranolol (Inderal) – 30–120

aOral dose or IM injection can be used.

Page 29 of 62Ovid: Applied Therapeutics: The Clinical Use Of Drugs

17/8/2010mk:@MSITStore:C:\Users\askari\Desktop\E%20Book\Applied\Applied%20Therapeuti...

Page 30: Schizophrenia

there are differences in adverse effects and duration of action. Trihexyphenidyl is the least sedating but more prone

to abuse, whereas diphenhydramine is the most sedating. Benztropine has the longest duration and can be used once

or twice a day if needed, whereas the others have to be used three to four times a day. Benztropine 1 mg twice a

day orally could be initiated for S.B. because he is likely too psychiatrically unstable to tolerate a reduction in the

haloperidol dose. The tremor, rigidity, and akinesia should begin to resolve within the first few days of treatment.

Alternatively, a switch to an atypical antipsychotic may negate the need for an antiparkinsonian drug in S.B.

14. Benztropine 1 mg BID is started one week later, all of S.B.'s acute symptoms of parkinsonism have

disappeared. How long should benztropine be continued in S.B. now that his EPS are resolved?

The long-term treatment of antipsychotic-induced parkinsonism with antiparkinsonian medication is controversial.154

The World Health Organization published a consensus statement recommending that the prophylactic use of

anticholinergic medication in patients receiving antipsychotics should be avoided or used only in cases where

alternative strategies have failed.157 For S.B., an attempt to taper and discontinue the antiparkinsonian treatment

should be initiated 6 weeks to 3 months after symptoms resolve. Unfortunately, as many as 30% of patients

chronically treated with typical antipsychotics continue to experience parkinsonian symptoms.90 If that should occur

with S.B., he should be switched to an atypical agent.

15. What risks are associated with long-term anticholinergic treatment?

The risks of anticholinergics include constipation, dry mouth leading to dental caries, and blurred vision. They can

also impair memory, especially in older patients. Some patients develop tolerance to these adverse effects, but

others do not, even with chronic use. Patients sometimes abuse anticholinergic medications for their mood-elevating

and hallucinogenic effects. Abuse may be confused with reluctance to discontinue antiparkinsonian medication

because of the fear of recurrent EPS or ongoing symptoms.

16. One week after the benztropine was started, S.B.'s psychiatric symptoms began to respond to the

haloperidol, but he developed acute urinary retention. Reduction of the benztropine dose to 1 mg/day did not

improve his urinary retention. What alternative treatments without anticholinergic effects are available to

manage S.B.'s pseudo-parkinsonian symptoms?

S.B. is just beginning to gain benefit from his haloperidol; therefore, the haloperidol dose should not be reduced.

Amantadine, an antiparkinsonian medication that directly stimulates postsynaptic dopamine receptors and restores

the cholinergic and dopaminergic balance in the nigrostriatum, would be a reasonable alternative to benztropine in

S.B. Amantadine is an option for patients who cannot tolerate or who respond poorly to anticholinergic drugs. It

often is preferred in the elderly because of the lower incidence of cognitive impairment. The parkinsonian symptoms

usually respond within 24 hours.

S.B. should be started on amantadine 100 mg twice a day and benztropine should be discontinued. He should be

monitored to determine whether EPS reappear and his urinary retention problem is corrected. S.B. also should be

monitored for the appearance of amantadine-associated, dose-related adverse effects such as tremor, slurred

speech, ataxia, depression, hallucinations, rash, orthostatic hypotension, and insomnia. If he cannot tolerate

amantadine, or if amantadine does not control his EPS, an atypical antipsychotic agent can be considered.

Akathisia

Akathisia is a syndrome consisting of subjective feelings of restlessness or the urge to move and an objective motor

component expressed as a semipurposeful movement most often involving the lower extremities (pacing, rocking,

and an inability to sit or stand in one place for extended periods of time). Akathisia is observed in up to 50% of

patients treated with typical antipsychotics and ranges from 5% to 15% of patients treated with atypical agents.58,158

Akathisia is often extremely distressing to patients, is a common cause of medication nonadherence and, if allowed

to persist, can produce dysphoria and possibly aggressive or suicidal behavior.58 It is often difficult to distinguish

from psychomotor agitation and worsening psychosis. Hence, the clinician must take care not to misdiagnose

akathisia because an increase in antipsychotic dose could worsen this adverse event. It usually develops within days

to weeks after initiating antipsychotic therapy.

The pathophysiology of akathisia is unclear, but much attention has focused on two theories.154 One theory states

that the mesocortical postsynaptic dopamine blockade leads to increased locomotor activity. An alternate theory

claims that akathisia is caused by dopamine antagonist-induced dysregulation of noradrenergic tracts that project

from the locus ceruleus to the limbic system.

17. J.P. was diagnosed with paranoid schizophrenia. His acute episode improved with trifluoperazine 15 mg HS,

and he had no other dystonic reactions. Two weeks later he became increasingly agitated, began pacing the

P.78p19

Page 30 of 62Ovid: Applied Therapeutics: The Clinical Use Of Drugs

17/8/2010mk:@MSITStore:C:\Users\askari\Desktop\E%20Book\Applied\Applied%20Therapeuti...

Page 31: Schizophrenia

floor, was unable to sit or lie down for longer than 10 minutes at a time, and subjectively had a feeling that he

described as “I have ants in my pants.” J.P. is observed rocking back and forth from one foot to the other while

standing in line for his dinner. He has no symptoms of antipsychotic-induced parkinsonism. Should the dose of

trifluoperazine be increased?

J.P.'s symptoms of rocking, pacing, agitation, and the inability to sit still are consistent with akathisia. Because his

psychiatric symptoms have improved and the new symptoms developed within 2 weeks after the initiation of

trifluoperazine, a diagnosis of akathisia is more probable than unresponsiveness to the antipsychotic medication.

Therefore, the trifluoperazine dose should not be increased because it can worsen the akathisia.

18. How should J.P.'s akathisia be managed?

Akathisia is less responsive to treatment than are antipsychotic-induced parkinsonism and dystonia.58 As an initial

approach to manage his akathisia, a small reduction (5 mg/day) in his trifluoperazine dosage should be attempted. If

symptoms of his psychosis return or if the akathisia persists, the addition of a β-blocking agent, an anticholinergic

drug, or a benzodiazepine can be considered.58,154 Propranolol would be a recommended agent because there is a

suggestion that its efficacy and safety profile may be preferable compared with other agents.159 Anticholinergic

drugs are appropriate alternatives if antipsychotic-induced parkinsonism is present. If his symptoms do not improve

in response to a dosage reduction of his trifluoperazine within 1 week, J.P. should be started on propranolol 20 mg

three times a day, and the dose should be increased by 20 mg every other day to a maximum of 120 mg, if

necessary.159 He has no contraindications to β-blocker therapy and does not have antipsychotic-induced

parkinsonism; therefore, antiparkinsonian agents (Table 78-12) are not preferred over propranolol. If the akathisia

persists after a trial of propranolol 120 mg/day therapy for 1 week, benzodiazepines should be tried or he should be

switched to an atypical antipsychotic.

Tardive Dyskinesia (TD)

TD is a syndrome characterized by involuntary choreoathetoid movements that occurs in individuals taking long-term

antipsychotics. The face (tics, blinking, grimacing), tongue (chewing, protrusion, tremor, writhing), lips (smacking,

pursing, puckering), neck and trunk (torsion and torticollis), and limbs (toe tapping, pill rolling, and writhing) are

commonly involved. The movements may be choreiform (rapid, jerky, nonrepetitive), athetoid (slow, sinuous,

continual), or rhythmic (stereotypic) in nature.

TD occurs in approximately 20% of patients who receive long-term treatment with typical agents.149 The cumulative

annual incidence is approximately 5% through the first 4 years of treatment in an adult who receives typical

antipsychotic treatment.160 Advancing age is the most consistently observed risk factor for the development of TD.161

The annual incidence rates of developing TD are three- to fivefold higher in older patients compared with younger adults.162,163 Other risk factors include higher mean daily and cumulative

antipsychotic doses, and presence of extrapyramidal signs early in treatment.162,163 Although there is some indication

that the risk of TD may be higher in women, nonwhites, patients with affective psychiatric disorders, and patients

taking concomitant anticholinergic agents, these findings have not been consistently observed.161

For most patients, TD does not seem to be progressive or irreversible; it can be reversed with discontinuation of the

antipsychotic agent. The onset of symptoms tends to be subtle with a fluctuating course.161 Most TD cases are

relatively mild. However, a portion of patients (5%–10%) may develop a form of TD that is severe enough to impair

functioning.164 Severe oral dyskinesia may result in dental and denture problems that can progress to ulceration and

infection of the mouth as well as muffled or unintelligible speech. Severe orofacial TD can impair eating and

swallowing, which in turn could produce significant health problems. Gait disturbances owing to limb dyskinesia may

leave patients vulnerable to falls and injuries.

Although the exact cause of TD is unknown, dopaminergic hypersensitivity, disturbed balance between dopamine

and cholinergic systems, dysfunction of GABAergic and noradrenergic systems and neurotoxicity via free radicals

have been proposed.149

19. C.M., a 31-year-old woman, was diagnosed with chronic paranoid schizophrenia 9 years ago. She responds

to antipsychotic drugs, but has been hospitalized six times because of nonadherence to her medications. She

has taken loxapine, haloperidol, and fluphenazine in the past and currently is being treated with

trifluoperazine 25 mg HS (decreased from 30 mg 1 month ago). C.M. also has been taking trihexyphenidyl 2 mg

TID for the past 5 years. Involuntary movements (including tongue protrusion, frequent blinking, and writhing

movements of her legs) were noted during a recent evaluation. What data in C.M.'s history are consistent with

TD?

The 9 years of antipsychotic treatment and the symptoms of tongue protrusion, blinking, and writhing leg

P.78p20

Page 31 of 62Ovid: Applied Therapeutics: The Clinical Use Of Drugs

17/8/2010mk:@MSITStore:C:\Users\askari\Desktop\E%20Book\Applied\Applied%20Therapeuti...

Page 32: Schizophrenia

movements are consistent with TD. Long-term anticholinergic treatment (trihexyphenidyl) also may contribute to

the development of C.M.'s abnormal movements.

20. C.M. has no history of abnormal movements and currently has no EPS. What disorders or medications can

produce symptoms similar to those of TD?

Tourette syndrome, dental problems, Huntington's or Sydenham's chorea, chorea of pregnancy, and systemic lupus

erythematosus are among the disorders that have been associated with dyskinetic movements. Medications such as

metoclopramide, amoxapine, bromocriptine, levodopa/carbidopa, and pramipexole165 also can cause TD.

Spontaneous dyskinesia resembles TD but can occur in patients without previous exposure to antipsychotic

medications.161 This form of abnormal movement is noticeably more common in elderly patients. A baseline

dyskinesia rating is essential before treating elderly patients with antipsychotic medications to avoid the potential

diagnostic dilemma of differentiating spontaneous dyskinesia from antipsychotic-induced TD.

TD varies in severity and presentation and is reversible in many cases; therefore, rating scales are needed to

standardize assessments (Table 78-11). A temporal relationship between an antipsychotic dose change and

movement severity should be evaluated when performing assessments. An increase in the antipsychotic dose can

clinically suppress the symptoms, whereas a decrease in dose can transiently unmask the movements often called

“withdrawal dyskinesia.”165 For example, the movements C.M. is exhibiting may have worsened as a result of the

recent dose reduction in trifluoperazine. On the other hand, a dose increase may produce a temporary lessening of

movement severity. Patients taking long-term antipsychotics should be evaluated for TD every 3 to 6 months using a

standardized rating scale (Table 78-11). Findings always should be documented in patient records to ensure

continuity of care.

21. C.M. is diagnosed with antipsychotic-induced TD. How should C.M.'s TD be managed?

Management of TD should focus first on prevention. That is, antipsychotic drugs must be reserved to treat conditions

known to respond (e.g., psychotic disorders such as schizophrenia, major depression with psychotic features,

schizoaffective disorder), and the total dose and duration of treatment should be minimized. Because C.M.'s long-

term exposure to antipsychotics is the likely cause of her TD, discontinuation of her trifluoperazine would be the

ideal treatment. Unfortunately, she has experienced multiple recurrent psychotic episodes and requires lifetime

treatment with antipsychotics. Because of the reduced TD liability with atypicals, C.M. should be switched to an

atypical agent.150,151,152,153 It has been recommended that an atypical antipsychotic be used for mild TD symptoms,

and that clozapine be considered when TD is severe or distressing to the patient.150,166 Rapid reduction in the dose of

trifluoperazine must be avoided to prevent severe withdrawal dyskinesias, but her dose should be slowly reduced as

an atypical antipsychotic is titrated to the lowest effective dose. This minimizes both her exposure to antipsychotic

medications and her corresponding dyskinesia risk. Trihexyphenidyl should also be discontinued because of its

possible contribution to TD. Because C.M. has already been diagnosed with TD, an evaluation of the risks and

benefits of continued antipsychotic treatment must be discussed with her and documented in the chart.

22. Are there other medications for treating T.D.?

A number of other agents have been studied for their potential therapeutic effects on TD. Drugs that augment GABA

neurotransmission (e.g., diazepam, clonazepam, valproic acid), adrenergic drugs (propranolol, clonidine) and free-

radical scavengers (vitamin E) have all been used, with limited or inconsistent results.160,167,168 These agents may be

useful as adjunctive treatments when TD persists despite the use of atypical agents.

Anticholinergic Effects

Anticholinergic effects are more significant with low-potency typical antipsychotics, and with the atypical agents,

clozapine and olanzapine. Clinically, patients complain of constipation, urinary retention, and dry eyes, mouth, and

throat. Dry mouth and throat can cause several additional problems, including dental caries and weight gain if thirst

is satisfied with high-sugar drinks. Oral fungal infections can also occur if gum or liquids with high sugar content are

regularly consumed. The most serious complications from medications with anticholinergic properties are delirium and adynamic ileus.169,170 Interestingly, clozapine causes significant

hypersalivation despite its anticholinergic effects. The mechanism is unknown, but it may be caused by clozapine's

augmentation of the adrenergic receptors that control salivation.171 Drug therapy with benztropine, amitriptyline,

and clonidine may alleviate hypersalivation; however, these agents are not effective in all patients and when they

are beneficial tolerance may develop. Although anticholinergic effects are a minor problem with high-potency

typical agents, some patients may still have problems when anticholinergic antiparkinsonian agents are added to

treat EPS.

P.78p21

Page 32 of 62Ovid: Applied Therapeutics: The Clinical Use Of Drugs

17/8/2010mk:@MSITStore:C:\Users\askari\Desktop\E%20Book\Applied\Applied%20Therapeuti...

Page 33: Schizophrenia

Cardiovascular Effects

Orthostatic Hypotension

Antipsychotic drugs can cause a variety of cardiovascular complications, but the most common problem is orthostasis

from α1-adrenergic blockade. Low-potency typical agents and atypical antipsychotics pose the greatest risk for

producing orthostatic hypotension. Orthostasis is most likely to occur during the first few days of treatment or when

increasing the dose of medication. Although tolerance usually occurs within 2 to 3 weeks, orthostasis necessitates

slow dose titration early in treatment for patients who are particularly prone to this side effect (e.g., the elderly).

Tachycardia

Tachycardia may occur as a result of the anticholinergic effects of antipsychotic medications on vagal inhibition, or

secondary to orthostatic hypotension.58 Clozapine produces the most pronounced tachycardia. If tachycardia is

sustained or becomes symptomatic, low doses of a β-blocker such as atenolol or propranolol can be useful once an

ECG has ruled out other medical causes.

Electrocardiographic Changes

ECG changes such as prolongation of the QT and PR intervals, ST-segment depression and T-wave flattening have

been observed with antipsychotics.172 The most clinically important of these potential changes is prolongation of the

QTc interval (which is the QT interval adjusted for heart rate) and has drawn the attention of the FDA. QTc

prolongation may lead to the development of ventricular tachyarrhythmias such as torsades de pointes and

ventricular fibrillation, which can cause syncope, cardiac arrest, or sudden cardiac death. All antipsychotics have

the potential to prolong the QTc interval to varying degrees. In 2000, Pfizer in consultation with the FDA, completed

a study in which the QTc intervals of patients taking several antipsychotics, at usual therapeutic dosages and in the

presence of a metabolic inhibitor, was compared (Fig. 78-2).173 Thioridazine was shown to prolong the QTc interval

at least 20 msec longer than haloperidol, risperidone, olanzapine, or quetiapine. This led to a boxed warning on the

FDA-approved product labeling stating that thioridazine has been shown to prolong the QTc interval in a dose-

related manner and its use should be limited to patients who cannot be managed on other antipsychotics. In the

same study, ziprasidone prolonged the QTc interval 5 to 15 msec longer than did haloperidol, risperidone,

olanzapine, or quetiapine. Unlike thioridazine, the ziprasidone effects were not dose related. The exact point at

which QTc prolongation becomes clinically dangerous is unclear. Because most reported cases of torsades de pointes

Figure 78-2 Comparison of QTc changes with antipsychotics. (Adapted from reference 173.)

Page 33 of 62Ovid: Applied Therapeutics: The Clinical Use Of Drugs

17/8/2010mk:@MSITStore:C:\Users\askari\Desktop\E%20Book\Applied\Applied%20Therapeuti...

Page 34: Schizophrenia

have appeared in individuals with a QT interval >500 msec, discontinuation of the suspected offending agent has

been recommended if the interval consistently exceeds 500 msec. It is important to note that no patients in this

study had QTc intervals exceeding 500 msec or arrhythmias. Despite the fact that no increased risk of arrhythmia or

sudden death has been demonstrated with ziprasidone, caution is warranted in patients with some types of cardiac disease and with an uncontrolled electrolyte disturbance. The coprescription of ziprasidone with other

drugs that prolong the QT interval should be avoided. Under most clinical circumstances, however, ziprasidone may

be safely used without ECG monitoring or other special precautions.

Metabolic Effects

Hyperprolactinemia

Antipsychotic-induced hyperprolactinemia has come to the forefront because of differences found among the

atypical agents. Before the introduction of the atypical agents, prolactin elevation was unavoidable because all

typical antipsychotics elevate serum prolactin by blocking the tonic inhibitory actions of dopamine in the

tuberoinfundibular tract.174 Among the atypical agents, risperidone and, to a lesser extent, olanzapine produce a

dose-related increase in prolactin levels that is equal to or greater than that seen with typical antipsychotics.175,176

The remaining atypical agents have little impact on serum prolactin levels.175 Hyperprolactinemia is of clinical

importance because it may lead to galactorrhea, gynecomastia, amenorrhea, anovulation, impaired

spermatogenesis, decreased libido and sexual arousal, and anorgasmia.177 Interestingly the hyperprolactinemia is not

always associated with clinical symptoms. For example, Kleinberg et al.176 failed to find an association between

sexual dysfunction and risperidone-associated hyperprolactinemia. This may be a product of the multifactorial

nature of sexual dysfunction or methodologic limitations in data collection. Patients often do not spontaneously

report symptoms of sexual dysfunction and clinicians must remember to ask about these potential side effects.175

Weight Gain

Weight gain is emerging as one of the most significant concerns associated with the use of antipsychotics,

particularly among the atypical agents. The mechanism of antipsychotic-induced weight gain is unclear, but

antagonism of histamine H1 and serotonin 5-HT2C receptors has been implicated.178 A genetic predisposition exists for

weight gain; a mutation in the 5-HT2c receptor gene may increase risk for weight gain from atypical antipsychotics.

No genetic tests exist to predict which patients will gain weight from the atypical antipsychotics.179,180 Another

theory is the potential decreased signaling of two serotonin receptors, 2A and 2C, increased calorie intake and

appetite, and/or decreased metabolic rate.180,181 The higher the binding affinity to the histaminergic receptor, the

more that agent is likely to be associated weight gain.180 Among the atypical agents, weight gain is most common

with clozapine and olanzapine, lowest with ziprasidone and aripiprazole, and intermediate with risperidone and

quetiapine.182 In a meta-analysis, Allison et al.183 found that the estimated weight gain at 10 weeks of therapy was

greater with clozapine (4.45 kg) and olanzapine (4.15 kg) relative to risperidone (2.1 kg) and ziprasidone (0.04 kg),

as illustrated in Figure 78-3. Moderate short-term weight gain has been demonstrated with quetiapine and is minimal

with aripiprazole.98,184 Similarly, clozapine and olanzapine have the largest associated weight gain with long-term

treatment.184 The weight gain observed with clozapine and olanzapine is not dose dependent and plateaus 6 to 12

months after treatment initiation.185,186 However, other authors suggest that antipsychotic-associated weight gain

plateaus within the first several months of treatment with risperidone, quetiapine, and ziprasidone can continue

over several years for clozapine and olanzapine.175 The issue of weight gain has important clinical implications in

light of the link with impaired glucose tolerance and type II diabetes, hyperlipidemia, and increased

mortality.58,187,188,189,190 Patients who had no weight gain due to atypical antipsychotics can still develop diabetes

mellitus. In a case review of 45 patients who developed or had worsening of their diabetes, nearly 50% had no

weight gain. Atypical antipsychotics may impair insulin sensitivity or glucose regulation independent of weight gain,

although the true reason for diabetes is still unknown.179,181

P.78p22

Page 34 of 62Ovid: Applied Therapeutics: The Clinical Use Of Drugs

17/8/2010mk:@MSITStore:C:\Users\askari\Desktop\E%20Book\Applied\Applied%20Therapeuti...

Page 35: Schizophrenia

Cardiovascular disease is the leading cause of death among patients with schizophrenia.191 On average, patients with schizophrenia live 20% shorter than the general population. They commit suicide up to 20 times more than the

general population; and patients with schizophrenia specifically aged 25 to 44 years old, are three times more likely

to die compared with the general population. This number has continued to increase as patients are no longer

institutionalized.179

23. L.A., a 45-year-old woman with chronic paranoid schizophrenia, had been managed successfully with

Figure 78-3 Comparison of weight change among antipsychotics. (Adapted from reference 183.)

P.78p23

Table 78-13 Monitoring Protocol for Atypical Antipsychoticsa

� Baseline Week 4 Week 8 Week 12 Quarterly Annually Every 5 years

Personal/Family history X � � � � X �

Weight (BMI) X X X X X � �

Waist circumference X � � � � X �

Blood pressure X � � X � X �

Fasting plasma glucose X � � X � X �

Fasting lipids profile X � � X � � X

aMore frequent assessments may be warranted based on clinical status.

Compiled from references 219 and 220.

Page 35 of 62Ovid: Applied Therapeutics: The Clinical Use Of Drugs

17/8/2010mk:@MSITStore:C:\Users\askari\Desktop\E%20Book\Applied\Applied%20Therapeuti...

Page 36: Schizophrenia

haloperidol for many years. She subsequently developed TD and was switched to olanzapine 20 mg HS. Before

initiation of olanzapine 4 months ago, L.A. was 54 and weighed 132 lb. She has responded well to the

olanzapine; however, she now weighs 148 lb. Can L.A.'s weight gain be attributed to olanzapine?

Weight gain has been reported with typical antipsychotics, but even greater weight gain has been seen with atypical

agents such as clozapine and olanzapine, and to a lesser extent, risperidone and quetiapine.182,183,184 The cause of

antipsychotic-induced weight gain is multifactorial and may include a change in food preferences, increased food or

fluid intake, carbohydrate craving, or a lack of activity. Blockade of the histamine and 5-HT2C receptors are also

contributory. H1 blockade causes sedation that may produce inactivity. H1 blockade may also increase weight by

interfering with normal satiety signals from the gut, resulting in overeating. Evidence for a role of 5-HT2C antagonism

is derived indirectly from appetite-suppressing drugs, such as fenfluramine, which are thought to act via 5-HT

agonism.192

L.A.'s weight gain should be taken seriously because it may contribute to medical conditions such as diabetes,

hyperlipidemia, coronary artery disease, gastrointestinal disorders, cancer, and hypertension.193,194 A significant

weight gain may cause her to have a poor self-image, which may lead to treatment nonadherence. Obese patients

with schizophrenia are 2.5 times more likely to discontinue their medication than nonobese patients.194 She should

be enrolled in a weight management program. If a patient gains 5% or more of their pretreatment body weight,

switching to another antipsychotic should be considered.195 For L.A., switching to another antipsychotic with lower

weight gain liability may be considered; however, this decision must be balanced against her current positive

response to olanzapine.

24. How should the potential metabolic complications associated with antipsychotics be monitored?

Although the risk of TD seems to be significantly reduced with the use of atypical antipsychotics, there is concern

regarding other long-term side effects with these agents including impaired glucose tolerance, type 2 diabetes, and

hyperlipidemia.175,187

L.A. should be informed about these potential complications. The routine monitoring of weight, fasting glucose

levels, and lipid panels have been recommended and appropriate therapeutic options should be initiated if

abnormalities are observed.175,187 Guidelines have been created for the monitoring of metabolic issues associated

with atypical antipsychotic use (Table 78-13). At baseline, a personal and family medical history should be obtained,

as well as height and weight (to determine body mass index), waist circumference, BP, fasting plasma glucose, and a

fasting lipid panel.195,196 Fasting blood glucose and lipid levels, as well as BP measurements, should be completed 3

months after drug initiation, then every year if the patient is within normal limits.195,196 Weight should be reassessed

at weeks 4, 8, and 12, then quarterly, after drug initiation.195,196 L.A. should be encouraged to self-monitor her

weight and report any significant weight fluctuations. Subjective evidence of a weight change may include a change

in clothes or belt size. In addition, she should be routinely monitored for the presence of diabetic symptoms (e.g.,

polyuria, polydipsia) at every clinic visit.

Neuroleptic Malignant Syndrome

25. C.B., a 25 year-old man, was hospitalized with the diagnosis of schizophrenia, paranoid type, and was

started on loxapine 25 mg HS. After 2 days of therapy, C.B. became rigid, seemed confused at times, and had a

fever of 41°C. A diagnosis of neuroleptic malignant syndrome (NMS) was made. What features of this syndrome

does C.B. have and how should it be treated?

NMS is a rare but potentially lethal adverse effect of antipsychotic therapy. The risk of NMS seems to be lower for

atypical agents than for typical agents.143 However, cases of NMS have been linked to treatment with clozapine,

risperidone, olanzapine, quetiapine, ziprasidone, and aripiprazole.197,198,199,200,201,202,203

NMS can occur hours to months after the initial drug exposure, and the mortality rate is reported to be as high as

20%.58 The incidence is estimated at between 0.02% and 3.23% of patients taking typical antipsychotic drugs.204 The

cardinal features include muscular rigidity, hyperthermia, autonomic dysfunction, and altered consciousness.

Extrapyramidal dysfunction (e.g., rigidity) and akinesia usually develop initially or concomitantly with a temperature

elevation as high as 41°C. Autonomic dysfunction includes tachycardia, labile BP, profuse diaphoresis, dyspnea, and

urinary incontinence. The patient's level of consciousness may vary from alert to mutism, stupor, and coma. Other neurologic findings

include sialorrhea, dyskinesia, and dysphagia. Symptoms usually develop rapidly over 24 to 72 hours. Creatine

kinase, CBC, and LFTs are usually increased.

C.B.'s fever of 41°C, rigidity, and confusion are consistent with NMS. C.B.'s loxapine should be discontinued and

supportive measures initiated to treat hyperthermia and prevent dehydration (e.g., antipyretics, a cooling blanket,

P.78p24

Page 36 of 62Ovid: Applied Therapeutics: The Clinical Use Of Drugs

17/8/2010mk:@MSITStore:C:\Users\askari\Desktop\E%20Book\Applied\Applied%20Therapeuti...

Page 37: Schizophrenia

and IV fluids).205 Secondary complications such as pneumonia and renal failure should be managed as they develop.

If C.B.'s condition does not show a trend toward improvement or worsens after 1 to 3 days of observation and

supportive therapy, a number of additional pharmacologic interventions should be considered.205 NMS has been

attributed to dopamine depletion caused by neuroleptic drug blockade of dopamine pathways in the basal ganglia

and hypothalamus. For that reason, dopamine agonists such as amantadine or bromocriptine sometimes are

beneficial. Dantrolene relaxes skeletal muscle and is specifically recommended for severe hyperthermia. C.B. should

be started on dantrolene 50 mg four times daily and either bromocriptine 2.5 mg three times daily or amantadine

100 mg three times daily to accelerate reversal of his condition.206 If he is unable to take oral medications, 1.25 to

1.5 mg/kg IV dantrolene should be used.

NMS is self-limiting and usually lasts 2 to 14 days after the oral antipsychotic is discontinued, or longer after

discontinuation of depot medications. C.B.'s response to therapy can be assessed by frequently monitoring his vital

signs and by measuring creatine kinase daily. After several weeks of recovery, treatment may be cautiously resumed

with another atypical antipsychotic.205,207

Sedation

Sedation can occur with any antipsychotic agent; however, it is most pronounced with the low-potency typical

agents and with clozapine and quetiapine. Sedation at the beginning of treatment may be desirable for anxious or

aggressive patients. However, persistent sedation during long-term treatment may adversely affect daily functioning

and quality of life. Most patients develop some tolerance to these sedating effects over time, but it can be

minimized by reducing the dose or shifting administration to bedtime.

Hepatic Dysfunction

26. A.S., a 24-year-old woman, is brought to the hospital because of unusual behavior and violence toward her

mother. A.S. is diagnosed with schizophrenia, paranoid type, acute exacerbation. Quetiapine 25 mg BID was

prescribed initially and the dose gradually increased until 200 mg TID was achieved. Her baseline laboratory

tests (CBC with differential, a chemistry profile that included LFTs, and serum electrolytes) were within normal

limits. The same tests were repeated 10 days after her psychosis was under evaluation. The aspartate

aminotransferase and alanine aminotransferase are now 2.5 times normal, but she has no GI complaints or

medical problems. Should quetiapine be discontinued at this time?

Benign elevations in LFTs (i.e., increases in aspartate aminotransferase or alanine aminotransferase less than two to

three times normal) have long been reported early in the course of therapy with most antipsychotic drugs.34,66,85

Such increases are rarely problematic; thus, no routine assessment of liver function in mandated by the FDA.

A more specific hepatic complication, cholestatic jaundice, has been associated with certain antipsychotic agents.

The phenothiazines, especially chlorpromazine, have been implicated. Most cases develop within the first month of

therapy and usually are preceded by prodromal symptoms of fever, chills, nausea, upper gastric pain, malaise, and

pruritus. Discontinuation of the phenothiazine and symptomatic care are the primary modes of treatment because

the cholestatic jaundice generally is self-limiting and usually resolves within 2 to 8 weeks. Occasionally, a more

chronic course may develop. Once the signs and symptoms have resolved, an alternate class of antipsychotic,

preferably a nonphenothiazine, should be prescribed.

The LFTs in A.S. are only modestly abnormal, and she is not experiencing symptoms of hepatotoxicity or cholestatic

jaundice. Therefore, quetiapine should be continued. Routine laboratory monitoring of LFTs does not prevent drug-

induced cholestatic jaundice; thus, no follow-up laboratory tests are required unless symptoms of hepatic

dysfunction develop. Awareness and evaluation of A.S. for prodromal symptoms is the most appropriate action at

this time.

Ocular Effects

27. Are there any additional baseline tests that should have been performed on A.S. before starting quetiapine?

The ocular effects of antipsychotic agents are well recognized and are generally of minor clinical consequence.

Corneal and lens changes have been reported with several phenothiazines and thiothixene. Chlorpromazine is the

most commonly implicated, with the risk being greatest with long-term, high-dose exposure (1–3 kg lifetime dose).208

If A.S. has not yet had a slit-lamp evaluation, an ophthalmologist should evaluate her as soon as possible. A 6-month

follow-up examination is also indicated. Quetiapine should probably be discontinued if lens changes were noted on

Page 37 of 62Ovid: Applied Therapeutics: The Clinical Use Of Drugs

17/8/2010mk:@MSITStore:C:\Users\askari\Desktop\E%20Book\Applied\Applied%20Therapeuti...

Page 38: Schizophrenia

any of the examinations.

Temperature Dysregulation and Dermatologic Effects

28. N.M., a 27-year-old man, recently was diagnosed with undifferentiated schizophrenia and was stabilized

with chlorpromazine 400 mg HS during a 3-month stay in an inpatient unit. N.M. has continued chlorpromazine

400 mg HS as an outpatient and has done so well that he is ready to return to work as a laborer at a

construction site. What precautions should he take when working outside?

N.M. should be advised to wear a hat when working outside, drink fluids, stay in the shade as much as possible, and

seek a cooler environment if he feels hot. Antipsychotics can cause temperature dysregulation, likely by inhibiting

hypothalamic temperature regulation. The net result is poikilothermia (the normal body temperature cannot

respond to heat or cold, and patients become hypothermic or hyperthermic, depending on the surrounding

temperature).209 The strong anticholinergic effects of N.M.'s chlorpromazine can impair cutaneous heat elimination

and further exacerbate the problem. Olanzapine and clozapine also have strong anticholinergic properties,

necessitating caution for patients exposed to excessive heat.

N.M. should also be advised to use a sunscreen with maximum sun protection factor, along with protective clothing

because chlorpromazine-induced photosensitivity can predispose him to severe sunburns. The tricyclic structure of

some antipsychotic drugs absorbs ultraviolet rays, producing free radicals that damage skin. Chlorpromazine is the

most common cause of photosensitivity, but it can occur with all phenothiazines and with thiothixene.209

Other dermatologic reactions can occur with antipsychotics, and N.M. should be advised to report any skin

abnormalities to his physician. Dermatitis, presenting with a maculopapular rash on the face, neck, and upper chest,

occurs in approximately 5% of patients shortly after starting chlorpromazine. Localized or generalized urticaria also

can develop. Antihistamines usually provide adequate relief, but the antipsychotic agent may have to be

discontinued in severe cases. The rash rarely reappears after resumption of treatment.

Seizures

29. R.A., a 26-year-old woman recently diagnosed with chronic paranoid schizophrenia, is brought to the

hospital after assaulting a neighbor. She presents in an acute psychotic state and has not calmed down since

entering the hospital. She has struck two staff members on the psychiatric unit. R.A. has had generalized tonic–

clonic seizures since age 14 and currently is taking carbamazepine 300 mg TID (serum concentration on

admission is 8.2 mg/mL). R.A. has not been taking any antipsychotic medications and has been seizure free for

1 year. She needs acute treatment with an antipsychotic because of her dangerous behavior. How should an

antipsychotic be initiated to treat her schizophrenia in light of her seizure disorder?

Antipsychotic drugs can lower the seizure threshold, producing seizures in patients who previously were seizure free.

Seizures are most common with low-potency typical antipsychotics and clozapine.58 Clozapine-induced seizures are

dose related. The seizure rate is approximately 1% at doses below 300 mg/day, 2.7% at doses between 300 and 600

mg/day, and 4.4% at doses above 600 mg/day.210 Because R.A. is currently a danger to herself and to others, the

benefits of antipsychotic treatment outweigh the risks. Strategies to minimize the risk of seizures should be

employed and include slow dose titration, use of lowest effective doses, and possible concurrent administration of

an antiepileptic drug (e.g. carbamazepine).211

Haloperidol, an agent with a low risk for causing seizures, is a good selection. An initial dose of 5 mg PO or 2.5 mg IM

should be given immediately, and additional dosages should be carefully administered until the psychosis resolves or

she is no longer dangerous. The decision to use scheduled or as-needed medications should be based on whether she

remains dangerous after a few immediate doses and the underlying cause of the psychosis. Supplemental

benzodiazepines may be used to avoid high doses of haloperidol. If necessary, R.A.'s carbamazepine dose should be

adjusted to maintain good seizure control and serum concentrations should be monitored. Carbamazepine (and other

enzyme-inducing antiepileptic drugs) can influence the hepatic metabolism of antipsychotic agents. R.A. should be

monitored for recurrent symptoms of schizophrenia if her carbamazepine dosage is increased.

Sexual Dysfunction

30. K.J., a 24-year-old man with chronic paranoid schizophrenia, was rehospitalized for an acute exacerbation

secondary to nonadherence with thioridazine 400 mg HS. During the medication history, it is discovered that he

stopped taking thioridazine because he lost his interest in sex. When he tries to have intercourse, he

experiences delayed ejaculation. About a week after stopping the thioridazine he is able to have a normal

ejaculation. How does thioridazine contribute to K.J.'s sexual dysfunction?

P.78p25

Page 38 of 62Ovid: Applied Therapeutics: The Clinical Use Of Drugs

17/8/2010mk:@MSITStore:C:\Users\askari\Desktop\E%20Book\Applied\Applied%20Therapeuti...

Page 39: Schizophrenia

Thioridazine is recognized as the most common cause of antipsychotic-induced sexual dysfunction.212 Sexual side

effects such as diminished libido, impaired arousal, and erectile and orgasmic dysfunction, however, have also been

reported with both other typical agents and atypical agents.175 The causes of antipsychotic-induced sexual

dysfunction are related to a number of factors including hyperprolactinemia via dopamine blockade, α-adrenergic

blockade, and anticholinergic and sedative effects.213 Impaired sexual function has also been observed in untreated

patients with schizophrenia, making the distinction between drug-induced and disease-induced symptoms

difficult.214 K.J.'s sexual dysfunction likely is related to the thioridazine because the symptoms resolved after he

stopped the medication. Lowering the dose of thioridazine or converting him to another antipsychotic with less

influence on sexual function may be helpful, although no reliable evidence is available to indicate which drugs are

least likely to cause sexual dysfunction.

Predictors of Medication Response As a general rule, antipsychotic agents should be initiated and titrated over the first few days to an average

effective “target” therapeutic dose unless the patient's physiological status or history indicates that this dose may

result in unacceptable adverse events.215 After 1 week on the “target” dose, a modest dosage increase (within the

recognized therapeutic range) may be considered if minimal or no improvement has been observed. In instances in

which symptoms improve after the dose is increased, it may be difficult to know whether the response resulted from

the dose change or from additional days on the drug. The duration of a therapeutic trial is 3 to 8 weeks in patients

with little to no response, and 5 to 12 weeks in patients with a partial response.166,215 Dosing recommendations for

the antipsychotics are provided in Table 78-10.

Factors to Consider When Evaluating a Poor Response A large proportion of patients with schizophrenia do not have an adequate response to antipsychotic therapy.

Inadequate response may be caused by inadequate dosing, poor adherence, or true resistance to antipsychotic

treatment.216 Some of these factors can be addressed. In some patients, the therapeutic antipsychotic dose cannot

be reached because of intolerable side effects. As mentioned, a therapeutic trial of 3 to 12 weeks at an optimal

dosage may be necessary to determine the full benefit from an antipsychotic agent. Nonadherence with medications

is a common obstacle in the management of schizophrenia, even in supervised settings. Blood level monitoring may

be helpful to identify nonadherent patients as well as those in whom a pharmacokinetic factor may account for a

poor response (e.g., poor drug bioavailability, rapid metabolizer, concomitant use of a metabolism-inducing agent). Therapeutic ranges of some antipsychotics are provided in

Table 78-9.

A single definition of treatment resistance does not exist, but factors to consider in this definition include chronic or

repeated hospitalizations, persistent positive symptoms, lack of improvement in negative symptoms, and

breakthrough symptoms despite adherence to treatment.216 Guidelines for determining treatment resistance have

been proposed and include at least two prior drug trials of 4 to 6 weeks duration at 400 to 600 mg chlorpromazine

(or equivalent, i.e., 8–12 mg of risperidone) with no clinical improvement, >5 years without a sustained period of

good social or occupational functioning, and persistent psychotic symptoms.216,217

Several strategies have been proposed if a change in antipsychotic medication is necessitated by side effects or

insufficient efficacy.140 These include an abrupt switch (abrupt cessation of the current drug, with abrupt

introduction of the new one at the expected therapeutic dose), a gradual switch (slow downward adjustment of the

dosage of the current medication, with slow upward adjustment of the dosage of the new drug) or an overlapping

switch (abrupt introduction of the new medication overlapping with the current medication, followed by downward

adjustment of the dosage of the previous medication). None of these strategies has been demonstrated to be

superior to another in terms of efficacy or safety, and the switch strategy depends on the clinical presentation.

31. M.S., a 24-year-old man, was brought to the ED by the police because voices told him to strike his mother.

His medical workup and urine drug screen were negative. M.S. has a diagnosis of schizophrenia,

undifferentiated type, and has been hospitalized four times in the last 2 years secondary to poor adherence to

his prescribed medication. He has had the same presentation on all previous hospitalizations. During his last

hospitalization, M.S. was stabilized on thiothixene (Navane) 20 mg orally at bedtime without any signs of

adverse effects. How can IM depot therapy decrease readmissions, and how do we determine if M.S. is a good

candidate for IM depot therapy?

M.S.'s history suggests that he is a good candidate for depot therapy. He responds to typical antipsychotics, but

relapses owing to nonadherence. He should be able to tolerate higher potency antipsychotics because he has

tolerated moderate doses of thiothixene. The patient should also be asked whether he is willing to take injections

P.78p26

Page 39 of 62Ovid: Applied Therapeutics: The Clinical Use Of Drugs

17/8/2010mk:@MSITStore:C:\Users\askari\Desktop\E%20Book\Applied\Applied%20Therapeuti...

Page 40: Schizophrenia

before proceeding with this strategy.

Conversion From Oral to Depot Therapy

32. How could M.S. be converted from oral therapy to depot therapy?

For patients such as M.S. who have never taken risperidone, haloperidol, or fluphenazine (the currently available

long-acting depot forms of antipsychotics), an oral trial lasting several days to weeks is needed before conversion.

The trial is necessary to ensure tolerability, evaluate response, and, when possible, determine the minimum

effective dose.

Various formulas to convert from oral formulations to the decanoates have been proposed, but no method has been

proved clinically superior. Clinicians should strive to use the longest dosing interval between injections and minimize

the duration of combination oral and depot therapy. Careful evaluation of both tolerance and response is required

for several months after initiating depot therapy.

Fluphenazine Decanoate Conversion

After stabilizing the patient on oral fluphenazine, multiply the total daily fluphenazine oral dose by 1.2 and

administer as fluphenazine decanoate IM every 1 to 2 weeks.218 Alternatively, 12.5 mg of fluphenazine decanoate

can be administered IM every 1 to 2 weeks for every 10 mg (rounding to the nearest 10-mg increment) of oral

fluphenazine per day (e.g., 25 mg/day PO rounded to 30 mg/day and given as 37.5 mg of decanoate).219 The

decanoate dosing interval may be increased to every 3 weeks after 4 to 6 weeks of therapy because of fluphenazine

accumulation. There are no specific guidelines for the continuation of oral therapy after initiating the depot

formulation; however, combination oral and IM therapy should be limited to the initiation period (1–4 weeks) or

during times of decompensation.

Haloperidol Decanoate Conversion

Elderly patients or those stabilized on <10 mg of oral haloperidol per day should receive haloperidol decanoate in an

IM dose that is 10 to 15 times the oral daily dose every 4 weeks. If higher oral doses are needed for stabilization,

then the first decanoate dose should be 15 to 20 times the oral daily dose to a maximum of 450 mg.220 The first

injection of haloperidol decanoate should not exceed 100 mg. If more is required, the balance of the dose can be

given in an additional one to two injections (provided that there are no EPS) over 3 to 7 days. Because haloperidol

accumulates, the monthly decanoate dose should be decreased every 3 to 4 months by 25% until a minimum

effective dose is achieved. Most patients can be effectively managed with a maintenance dose of 50 to 200 mg every

4 weeks.166 Guidelines for concomitant oral therapy remain unclear and should be based on response and adverse

effects, with the goal of depot alone after the first month of treatment if the aforementioned strategy is used.218

Using these guidelines, M.S. should be converted from thiothixene 20 to 10 mg of oral haloperidol per day (Table 78-

10). Oral haloperidol should be continued alone for at least 1 week, until M.S. is stabilized. Once stabilization with

oral haloperidol has occurred, haloperidol decanoate 100 mg IM can be administered. This can be followed with 50

mg IM 3 days later if no adverse effects are noted after the first injection. The total initial dose is 150 mg every 4

weeks (15 times the oral dose). A 200-mg total dose is also acceptable if the clinician believes the patient needs

more drug. No oral medication is needed after the second injection and should only be given if M.S. shows signs of

decompensation. M.S. should be monitored carefully for extrapyramidal side effects and response. M.S. should

receive haloperidol decanoate 150 mg IM every 4 weeks with the dose adjusted according to response and tolerance;

the dose can be decreased after 3 to 4 months because of drug accumulation.

Long-Acting Risperidone Formulation

It is recommended that patients establish tolerability with oral risperidone prior to initiating treatment with

Risperdone CONSTA. In published clinical trials, IM doses of 25, 50, or 75 mg have been administered at 2-week

intervals.141,142 The recommended dose is 25 mg IM every 2 weeks. Oral therapy of risperidone should be continued for 2 to 3 weeks

after the first IM injection and then discontinued.

Inadequate Response

33. J.H., a 33-year-old man, was diagnosed with chronic paranoid schizophrenia at age 21 and has been

hospitalized 15 times because of his illness. He repeatedly presents in a paranoid and hostile state complaining

of auditory hallucinations and paranoid delusions in which the police and the people in his apartment building

P.78p27

Page 40 of 62Ovid: Applied Therapeutics: The Clinical Use Of Drugs

17/8/2010mk:@MSITStore:C:\Users\askari\Desktop\E%20Book\Applied\Applied%20Therapeuti...

Page 41: Schizophrenia

are trying to kill him. His symptoms have prevented him from obtaining employment, and he has no close

friends. He has been treated with fluphenazine 20 mg HS and perphenazine 48 mg HS, but has had only a

partial response to each medication. After each discharge, J.H. stays out of the hospital for approximately 6

months, then relapses, even with good adherence. He currently is taking haloperidol 15 mg HS with only

minimal improvement after 2 months of treatment. What can be done to improve his response?

J.H. meets criteria for treatment-resistant schizophrenia, and clozapine is the “gold standard” for the treatment of

this type of patient.216 However, considering the risk of agranulocytosis, the burden of side effects, and the

requirement of white blood cell (WBC) monitoring associated with clozapine therapy, one of the other atypicals

should be tried before proceeding to clozapine.166,216

Clozapine

Indications

34. J.H. did not respond to an adequate trial of quetiapine 400 mg BID and was just admitted to the hospital

again. He is becoming increasingly isolated and paranoid. He has no other medical conditions and is taking only

lorazepam 1 to 2 mg Q 4 to 6 hr PRN. His last dose of quetiapine was 2 days ago. Because of his refractory

illness, J.H. is being considered for clozapine therapy. What are the advantages and disadvantages of clozapine

therapy? What makes him a good candidate for clozapine?

Clozapine is approved by the FDA for treatment of resistant schizophrenia or when adverse effects such as TD and

EPS preclude use of other antipsychotics. Clozapine is contraindicated in patients with a history of a

myeloproliferative disorder, uncontrolled epilepsy, paralytic ileus, clozapine-induced agranulocytosis or

granulocytopenia a current WBC count below 3,500 mm3 or ANC count below 2,000/mm3. Clozapine also should be

used with caution in patients who cannot tolerate anticholinergic effects, in those at risk for drug-induced

orthostasis, or in patients with significant renal or hepatic disease.221 Because of J.H.'s progressive decline in social

functioning and inadequate response to haloperidol, fluphenazine, perphenazine, and quetiapine, clozapine should

be considered.

Clozapine has clearly demonstrated superiority to typical agents in treating refractory schizophrenia. In the pivotal

study by Kane et al.,222 30% of clozapine-treated subjects versus 4% of chlorpromazine-treated subjects met criteria

for response at 6 weeks. A review and meta-analysis of seven controlled trials compared clozapine with typical

antipsychotics in treatment-resistant schizophrenia and found that clozapine was superior in terms of overall

therapeutic response, EPS, and adherence rate.223 However, similarly favorable response rates have not yet been

seen with other atypical agents in treatment resistant schizophrenia. Risperidone and olanzapine have been most

often studied in this regard. In a double-blind trial comparing risperidone with haloperidol in treatment-refractory

schizophrenia, a significantly higher response rate favoring risperidone was observed at week 4 but not at week 8.224

In a 6-week clinical trial, only 7% of patients prospectively determined to be resistant to haloperidol responded to

olanzapine, a rate that did not differ from chlorpromazine.225 In general, atypical agents other than clozapine have

not been consistently found to be as effective in treatment-resistant schizophrenia.226 Nevertheless, given the

problems associated with clozapine (risk of agranulocytosis, burden of other side effects, and the requirement for

hematologic monitoring), atypical agents should be tried before proceeding to clozapine in most patients.216

However, some patients do not respond to clozapine after an adequate trial of at least 6 months at a therapeutic

dose. In such situations, it is reasonable to prescribe the antipsychotic to which they had the historic best response.

Augmentation strategies, such as the addition of mood stabilizers (lithium and valproate), benzodiazepines,

propranolol, antidepressants, or another antipsychotic agent, may be attempted. However, evidence for this

approach is lacking.226 Treatment guidelines recommend reserving augmentation strategies for patients who fail or

cannot tolerate clozapine, although in practice these strategies are often tried before a trial of clozapine.58,227

Initiation of Therapy

35. How should clozapine be initiated in J.H.? What are its side effects and how should he be monitored?

Whenever possible, clozapine should be initiated when a patient is medication free. However, if the patient is too ill

to be taken off all antipsychotic drugs before adding clozapine, the current medication can be discontinued after a

therapeutic clozapine dose has been achieved. Alternatively, clozapine can be titrated upward while the first

antipsychotic is slowly tapered downward. Whenever possible, high-potency antipsychotics should be the concurrent

agent used during the clozapine titration to minimize additive adverse effects such as sedation, anticholinergic

effects, and orthostasis. Benzodiazepines should not be used for early behavioral and anxiety control, because their

combined use with clozapine can lead to respiratory or cardiac arrest.58

Page 41 of 62Ovid: Applied Therapeutics: The Clinical Use Of Drugs

17/8/2010mk:@MSITStore:C:\Users\askari\Desktop\E%20Book\Applied\Applied%20Therapeuti...

Page 42: Schizophrenia

Clozapine can be started in an outpatient setting if patients are carefully monitored for tolerability, especially

orthostasis. The starting dose is 12.5 to 25 mg once or twice a day with increases of 25 to 50 mg/day until a target

dose is reached. Clozapine is given in divided doses. The optimal dose has not been specifically delineated although

an initial target of 300 to 450 mg/day is reasonable based on clinical experience. The maximum daily dose is 900

mg, but titration should proceed slowly to such high doses, and only after adequate trials at lower doses. Response

is based on improvement of individual target symptoms and can be monitored by psychometric rating scales (Table

78-11). In addition, the overall ability of a patient to function and care for oneself during clozapine therapy also

should be considered.228 Most patients improve significantly during the first 6 weeks, but some take longer. Although

the exact length of an appropriate trial is unclear, 12 weeks is adequate for most clients; however, improvement may continue for 6 to 12 months after initiating therapy.58,229,230

Clozapine has been issued five blackbox warnings from the FDA. These included agranulocytosis, seizures,

myocarditis, increased mortality in elderly patients with dementia-related psychosis, and other adverse

cardiovascular and respiratory effects.231 The risk of seizures is dose dependent. The risk of fatal myocarditis is most

frequent in the first month of treatment with clozapine. Other adverse cardiovascular and respiratory effects

indicates orthostatic hypotension that may or may not be accompanied by syncope. Rarely, collapse with respiratory

or cardiac arrest may occur. This is why it is essential to start patients on 12.5 mg once or twice daily, whether it is

initiation of treatment, or the patient has been off of clozapine for 2 or more days.231

Clozapine causes agranulocytosis in approximately 1% of patients; therefore, baseline and weekly CBCs are required

during treatment. Clozapine is automatically discontinued if a patient is noncompliant with the blood work. After 6

months of continuous, stable treatment, patients can be changed to biweekly WBC monitoring if their WBC counts

are acceptable (≥3,500 mm3 and the absolute neutrophil count [ANC] is ≥2,000 mm3). After an additional 6 months

on a continuous, stable regimen, the patient may be changed to WBC monitoring every 4 weeks.231 If a patient takes

clozapine for <6 months with no problems, but stops the drug for <1 month, WBC monitoring can restart where it

was left off for a total of 6 months. If the break is >1 month, then an additional 6 months of weekly testing are

needed.

If baseline laboratory work is within normal limits and an informed consent is obtained, J.H. can start on clozapine

25 mg at bedtime. His clozapine dose should be increased by 25 to 50 mg every day until the dose is 300 mg/day,

the minimum dose at which most patients respond. The doses should be split and given as 150 mg twice a day.

Subsequently, clozapine should be increased incrementally according to the guidelines described. If J.H. has no

response to 300 mg after 4 to 8 weeks (based on a reduction in Brief Psychiatric Rating Scale scores and global

improvement), then the dose can be increased to 450 mg (150 mg TID). If response still is inadequate, then the dose

can be increased slowly to a maximum of 900 mg/day. Careful monitoring for orthostasis, hyperthermia, and

oversedation is needed, especially during early titration. Serum concentration monitoring can also help to guide dose

increases.

Clozapine may also cause sialorrhea, or hypersalivation. This is most often worst at night, and may lead to social

withdrawal, choking, or aspiration pneumonia.232 The incidence ranges from 31% of patients (premarket studies) to

up to 80% in several clinical trials.233 Several case reports have indicated that the scopolamine patch (1.5 mg/72

hour), ipratropium sublingual spray (0.03% or 0.06% given 2 sprays up to three times daily), atropine 1% ophthalmic

solution (2 drops swish and swallow twice daily), botulinum toxin (150 IU injected into each parotid gland), and

guanfacine (1 mg every morning) are effective in reducing, if not eliminating, sialorrhea.232,233,234,235,236 Other

medications commonly used are clonidine, anticholinergics (biperiden, benztropine, and diphenhydramine),

amitriptyline, terazosin, and propranolol.235,237

36. Four weeks after starting clozapine, J.H.'s WBC count drops to 4,000/mm3 from a baseline of 6,800/mm3.

Two weeks later, he has a WBC count of 2,100/mm3 and an ANC of 980/mm3. What action, if any, should be

taken?

[SI unit: WBC count, 4.0 and 2.1 × 109/L, respectively]

The hematologic adverse effect profile for clozapine is very different from that of other antipsychotic agents. The

incidence of clozapine-induced agranulocytosis is at least 10 to 20 times greater than the incidence with typical

antipsychotic medications.221 The incidence increases with age and is higher among women.238 Analysis of data from

150,409 patients followed by the Clozaril National Registry found agranulocytosis in 585 patients resulting in 19

deaths.238 The risk of agranulocytosis is greatest early in treatment with most cases appearing during the first 3

months of treatment. After 6 months, the risk for agranulocytosis decreases substantially but may occur in rare

instances up to 5 years into treatment.228,231 Trends in the weekly tests showing a continual decline in WBC count,

even if it remains >3,000/mm3, require careful attention. Guidelines for responding to reduction in WBC are

P.78p28

Page 42 of 62Ovid: Applied Therapeutics: The Clinical Use Of Drugs

17/8/2010mk:@MSITStore:C:\Users\askari\Desktop\E%20Book\Applied\Applied%20Therapeuti...

Page 43: Schizophrenia

described in Table 78-14.

Clozapine should be discontinued immediately, J.H. should be hospitalized, and hematology and infectious disease specialists should manage his care. Granulocyte colony-stimulating factor has been used successfully to manage

clozapine-induced agranulocytosis.239 Clozapine should not be reinstituted in J.H. because he has a very low ANC.

However, it should be noted that agranulocytosis can be reversible upon discontinuation of clozapine, in most cases,

with no persisting hematologic sequelae.238

P.78p29

Table 78-14 Guidelines for Response to Clozapine-Induced White Blood Cell Abnormalities

Do Not Initiate Clozapine

Initial WBC count <3,500/mm3

Initial ANC <2,000/mm3

History of myeloproliferative disorder

History of agranulocytosis or granulocytopenia related to clozapine

WBC <2,000/mm3 and/or ANC <1,000/mm3

Discontinue clozapine immediately

Do not rechallenge patient

Monitor WBC and differential: until normal and for at least 4 weeks:

—Daily until WBC > 3,000/mm3 and ANC > 1,500/mm3

—Twice weekly until WBC > 3,500/mm3 and ANC > 2,000/mm3

—Weekly after WBC > 3,500/mm3

Consider bone marrow aspiration

Protective isolation initiated if deficient granulopoiesis

WBC 2,000–3,000/mm3 or ANC 1,000–1,500/mm3

Discontinue clozapine

Monitor WBC and differential:

Page 43 of 62Ovid: Applied Therapeutics: The Clinical Use Of Drugs

17/8/2010mk:@MSITStore:C:\Users\askari\Desktop\E%20Book\Applied\Applied%20Therapeuti...

Page 44: Schizophrenia

37. What pharmacotherapy options could be tried to improve J.H.'s response now that clozapine is

contraindicated?

Sequential monotherapy trials with other atypical agents that he had not been previously exposed to may be

initiated. Alternatively, strategies such as the combination of an antipsychotic and a mood stabilizer or another

antipsychotic have been attempted. Because J.H. has no precautions or contraindications to an adjunctive trial of

lithium, he is a candidate for a 1-month lithium trial along with an antipsychotic medication other than clozapine.

The antipsychotic is selected after reviewing his medication history and choosing the agent to which he had the best

previous response. Lithium should be given at doses to achieve a target level of 0.8 to 1.2 mEq/L and continued for

1 month to allow the best chance for response. If the patient is unchanged at the end of 1 month, the lithium should

be discontinued and another adjunctive agent tried for a 1-month trial.

Serum Concentration Monitoring

38. E.H., a 68-year-old woman who was diagnosed with chronic undifferentiated schizophrenia >30 years ago.

She requires maintenance therapy with antipsychotic drugs because she relapsed when her haloperidol dose

was decreased to 10 mg/day. Currently, she is responding poorly to haloperidol 15 mg HS for 6 weeks despite

good adherence to her therapy. She has moderate pseudoparkinsonism controlled by benztropine 1 mg BID.

Why should a serum concentration of haloperidol be obtained before increasing E.H.'s dose of this drug?

Antipsychotic drug serum concentrations are not part of routine clinical care, but can be considered under the

following circumstances240:

� Poor response to moderate doses after an adequate trial (dose and duration)

� Serious or unexpected adverse effects at moderate dosages

� Deterioration in a compliant and previously stable patient

—Daily until WBC > 3,000/mm3 and ANC > 1,500/mm3

—Twice weekly until WBC > 3,500/mm3 and ANC > 2,000/mm3

May restart clozapine when:

—if no symptoms of infection present

—WBC > 3,500/mm3 and ANC > 2,000/mm3

WBC Drops to 3,000–3,500/mm3; >3,000/mm3 Over 1–3 Weeks; Immature WBCs Present

Repeat WBC with differential

If repeated WBC between 3,000 and 3,500/mm3 and ANC <2,000/mm3

—repeat WBC with differential until WBC > 3,500/mm3 and ANC >2,000/mm3

ANC, absolute neutrophil count; WBC, white blood cell.

Compiled from references 58 and 231.

Page 44 of 62Ovid: Applied Therapeutics: The Clinical Use Of Drugs

17/8/2010mk:@MSITStore:C:\Users\askari\Desktop\E%20Book\Applied\Applied%20Therapeuti...

Page 45: Schizophrenia

� Use of higher than usual dosages

� Evaluating whether the lowest effective dose is used in maintenance therapy

� Children, elderly, and medically compromised patients with potentially altered pharmacokinetics

It is unnecessary and not cost effective to obtain serum concentrations for antipsychotic drugs that do not have

defined therapeutic ranges. Most studies describe haloperidol's therapeutic range to be between 4 and 12

ng/mL.241,242 The recommended ranges for fluphenazine and thiothixene are 0.2 to 2.8 ng/mL and 2 to 15 ng/mL,

respectively.243,244,245 Evaluation of clozapine trials suggest an optimal range of 350 to 420 ng/mL.246,247 Greater

response rates were observed in olanzapine-treated patients whose plasma concentrations were >23.2 ng/mL in

blood samples collected 12 hours after the dose.248 Samples should be collected 5 to 7 days after a fixed dose and 10

to 12 hours after the last dose for oral preparations. Two to three months should elapse before samples are

collected in patients taking depot preparations.240

E.H. has received an adequate trial of haloperidol and has moderate antipsychotic-induced parkinsonism. To prevent

exposure to higher than necessary doses, a serum haloperidol concentration is warranted before increasing the dose.

The target serum concentration should be between 4 and 12 ng/mL.

Treatment Adherence Poor adherence with prescribed antipsychotic therapy is a significant problem in the long-term management of

schizophrenia. Comprehensive reviews suggest that deviation (primarily underuse) from prescribed regimens occurs

in about 50% of patients with schizophrenia.249,250 Recent advances in medication treatments with atypical agents

have produced fewer side effects, which some studies have shown has increased adherence.251,252 During any phase

of illness, nonadherence with maintenance antipsychotic therapy places patients at risk for exacerbation of

psychosis, increased clinic and emergency room visits, and rehospitalization.253,254 Nonadherence to prescribed

regimens can also compromise patients' daily functioning and quality of life. Long-acting depot antipsychotics should

be strongly considered for patients who have a history of nonadherence. In addition to assured medication delivery,

another benefit of depot preparations is avoidance of potential bioavailability problems. Clinicians should suspect

poor absorption if serum concentrations are much lower than expected for a given oral dose; however, factors such

as nonadherence or drug–drug interactions must also be ruled out. Disadvantages of long-acting depot formulations

include the time required to reach optimal dosing and the inability to immediately withdraw the drug if unpleasant

side effects develop.58,255 Because of this, patients should be converted to a depot form after confirmation that the

oral dosage form is safe and effective.

Considerations in specific populations

Pregnancy 39. D.M., a 26-year-old woman, has a 6-year history of chronic undifferentiated schizophrenia that has been

treated with chlorpromazine 400 mg HS. She is psychiatrically stable and was last hospitalized 2 years ago.

When D.M. decompensates, she isolates herself, is unable to care for herself, does not eat properly, and does

not maintain her grooming and hygiene. She just found out that she is 8 weeks pregnant. Should her

chlorpromazine be continued and what are the risks if the medication is continued?

Most of the available evidence regarding the use of antipsychotics during pregnancy has focused on typical

antipsychotics. Among the typical antipsychotics, pooled results of large retrospective and small prospective

controlled studies suggest an increased risk of congenital anomalies in patients treated with low-potency

phenothiazines, particularly when treated during the first trimester.256 Guidelines have recommended that the use

of antipsychotic medication during the first trimester should be minimized or avoided if possible, especially between

weeks 6 and 10.58 If an antipsychotic is necessary during this period, high-potency typical antipsychotics may be safer.58 In addition to their teratogenic effects, there are other reasons to avoid low-potency

antipsychotics. Low-potency agents can worsen constipation and cause orthostatic hypotension and uteroplacental

insufficiency. Guidelines have also recommended that medications should be tapered 1 week before delivery to

minimize neonatal complications such as dystonia, withdrawal dyskinesias, temperature dysregulation, and

irritability.58

Medication management should balance the needs of D.M. against those of her unborn child. D.M.'s schizophrenia has

been stable for 2 years. Chlorpromazine should be discontinued because it is a low-potency phenothiazine and D.M.

is currently in her first trimester. Because she is a danger to herself when ill, it is critical that treatment be initiated

at the first evidence of decompensation. D.M. should be monitored by her clinician, family members, and

P.78p30

Page 45 of 62Ovid: Applied Therapeutics: The Clinical Use Of Drugs

17/8/2010mk:@MSITStore:C:\Users\askari\Desktop\E%20Book\Applied\Applied%20Therapeuti...

Page 46: Schizophrenia

caseworker at frequent intervals for reemergence of psychotic symptoms. If D.M. decompensates, treatment should

be restarted with a nonphenothiazine, high-potency agent at the lowest possible dose (e.g., haloperidol or

fluphenazine 2–5 mg at night). If D.M. does not respond to low-dose haloperidol or fluphenazine, or if she develops

intolerable extrapyramidal side effects, then a lower potency antipsychotic drug or an atypical agent can be

selected. D.M.'s chart should include the following documentation: specific behaviors that necessitate treatment

(danger to self, fetus, or others), goals of therapy (acute control of dangerous behavior or remission), dose, route of

administration, schedule, and projected duration of treatment, obstetric history and other medications used during

pregnancy, alcohol or illegal drug use; regular progress notes on response and need for continued treatment; and

informed consent from the patient or guardian.

Children 40. M.J. is the 5-year-old son of a close friend of the family who was diagnosed with autism. His mother, R.J.,

knows you are a psychiatric pharmacist and asks your opinion about the use of atypical antipsychotics in

children with autism. What can you tell her?

Risperidone is the only atypical antipsychotic agent that is approved by the FDA for the symptomatic treatment of

irritability in autistic children and adolescents.99 In a study of 5- to 17-year-old children with autism accompanied by

severe tantrums, aggression, or self-injurious behavior, the children were randomly assigned to flexible doses of

either placebo or risperidone treatment (dose range, 0.5–3.5 mg/day) for 8 weeks.257 A significantly greater

percentage of children receiving risperidone responded to treatment than those receiving placebo (56.9% vs. 14.1%

respectively, p <0.001); 69% of the risperidone group had at least a 25% improvement on the irritability subscale of

the Aberrant Behavioral Checklist as rated by a parent or primary caretaker and confirmed by a clinician and a rating

of much or very much improved on a global improvement scale compared with only 12% in the placebo group (p

<0.001). The benefit from risperidone treatment was maintained for six months in two-thirds of the risperidone

responders. The overall reduction in the level of irritability was 57% at 8 weeks of risperidone in comparison with a

reduction of only 14% with placebo (p <0.001). Troost et al.258 evaluated the long-term effects of risperidone for an

additional 24 weeks after the initial 8-week open-label trial. Risperidone was effective in reducing disruptive

behavior in about one-half of the children, although considerable weight gain was observed.

Data with other atypical antipsychotics for the treatment of autism are limited. Olanzapine has the most data to

date with open-label trials suggesting efficacy; quetiapine, ziprasidone, and anpiprazole may also be

effective.259,260,261,262,263,264,265 Hollander et al.120 found olanzapine to be a potential alternative to risperidone for

improving global functioning of pervasive developmental disorders but the risk of significant weight gain may limit

its use in children/adolescents. In contrast, ziprasidone was not associated with significant weight gain.264

References

1. Piccinelli M, Gomez Homen F. Gender Differences in the Epidemiology of Affective Disorders and

Schizophrenia. Geneva: World Health Organization; 1997:61.

2. Murray CJL, Lopez AD, eds. Global Burden of Disease. Cambridge, MA: Harvard University Press; 1996.

3. Wu EQ et al. The economic burden of schizophrenia in the United States 2002. J Clin Psych 2005;66:1122.

4. Rupp A, Keith SJ. The costs of schizophrenia. Assessing the burden. Psychiatr Clin North Am 1993;16:413.

5. National Advisory Mental Health Council. Health care reform for Americans with severe mental illnesses:

report of the National Advisory Mental Health Council. Am J Psychiatry 1993;150:1447.

6. Rosenheck RA. Outcomes, Costs, and Policy Caution: A Commentary on the Cost Utility of the Latest

Antipsychotic Drugs in Schizophrenia Study (CUtLASS 1). Arch Gen Psych 2006;63:1074.

7. Andreasen NC, Carpenter WT. Diagnosis and classification of schizophrenia. Schizophr Bull 1993;19:199.

Page 46 of 62Ovid: Applied Therapeutics: The Clinical Use Of Drugs

17/8/2010mk:@MSITStore:C:\Users\askari\Desktop\E%20Book\Applied\Applied%20Therapeuti...

Page 47: Schizophrenia

8. Seaton BE et al. Sources of heterogeneity in schizophrenia: the role of neuropsychological functioning.

Neuropsychol Rev 2001;11:45.

9. Kendler KS, Diehl SR. The genetics of schizophrenia: a current genetic-epidemiologic perspective. Schizophr

Bull 1998;19:261.

10. Kendler KS, Robinett CD. Schizophrenia in the National Academy of Sciences-National Research Council

Twin Registry: a 16-year update. Am J Psychiatry 1983;140:1551.

11. American Psychiatric Association. Diagnostic and Statistical Manual of Mental Disorders Text Revision, 4th

ed. (DSM-IV-TR). Washington, DC: American Psychiatric Press; 2000.

12. Pearlson GD. Neurobiology of schizophrenia. Ann Neurol 2000;48:556.

13. Pulver AE. Search for schizophrenia susceptibility genes. Biol Psychiatry 2000;47:221.

14. Crow T et al. Schizophrenia as an anomaly of development of cerebral asymmetry: a postmortem study and

a proposal concerning the genetic basis of the disease. Arch Gen Psychiatry 1989;46:1145.

15. Andreasen NC et al. Magnetic resonance imaging of the brain in schizophrenia. Arch Gen Psychiatry

1990;47:35.

16. Gur R et al. Magnetic resonance imaging in schizophrenia: I. Volumetric analysis of brain and cerebrospinal

fluid. Arch Gen Psychiatry 1991; 48:407.

17. Kelsoe JR et al. Quantitative neuroanatomy in schizophrenia: a controlled magnetic resonance imaging

study. Arch Gen Psychiatry 1988;45:533.

18. Suddath RL et al. Temporal lobe pathology in schizophrenia: a quantitative magnetic resonance imaging

study. Am J Psychiatry 1989;146:464.

19. Suddath RL et al. Anatomical abnormalities in the brains of monozygotic twins discordant for

schizophrenia. N Engl J Med 1990;322:789.

20. Harrison PJ. On the neuropathology of schizophrenia and its dementia: Neurodevelopmental,

neurodegenerative, or both? Neurodegeneration 1995;4:1.

21. Bogrest B et al. Hippocampus-amygdala volumes and psychopathology in chronic schizophrenia. Biol

Psychiatry 1991;33:239.

22. Roberts G. Is there gliosis in schizophrenia? Investigation of the temporal lobe. Biol Psychiatry

1987;22:1459.

23. Kindermann SS et al. Review of functional magnetic resonance imaging in schizophrenia. Schizophr Res

1997;27:143.

24. Weinberger DR et al. Physiological dysfunction of dorsolateral prefrontal cortex in schizophrenia, I:

Page 47 of 62Ovid: Applied Therapeutics: The Clinical Use Of Drugs

17/8/2010mk:@MSITStore:C:\Users\askari\Desktop\E%20Book\Applied\Applied%20Therapeuti...

Page 48: Schizophrenia

Regional cerebral blood flow evidence. Arch Gen Psychiatry 1986;43:114.

25. Stevens AA et al. Cortical dysfunction in schizophrenia during auditory word and tone working memory

demonstrated by functional magnetic resonance imaging. Arch Gen Psychiatry 1998;55:1097.

26. Mubrin Z et al. Regional cerebral blood flow patterns in schizophrenic patients. Cerebral Blood Flow Bull

1982;3:43.

27. Andreasen NC et al. “Cognitive dysmetria” as an integrative theory of schizophrenia: a dysfunction in

cortical-subcortical-cerebellar circuitry? Schizophr Bull 1998;24:203.

28. Herz MI, Marder SR. Neurobiology. In: Schizophrenia: Comprehensive Treatment and Management.

Philadelphia: Lippincott Williams & Wilkins; 2002:3.

29. Andreasen NC et al. Defining the phenotype of schizophrenia: cognitive dysmetria and its neural

mechanisms. Biol Psychiatry 1999;46:908.

30. Weinberger DR. Neurodevelopmental perspectives on schizophrenia. In: Bloom FE, Kupfer DJ, eds.

Psychopharmacology: The Fourth Generation of Progress. New York: Raven Press; 1995.

31. Green MF, Neuchterlein KH. Should schizophrenia be treated as a neurocognitive disorder? Schizophr Bull

1999;25:309.

32. Creese I et al. Dopamine receptor binding predicts clinical and pharmacological potencies of

antischizophrenic drugs. Science 1976;192:481.

33. Seeman P et al. Antipsychotic drug doses and neuroleptic/dopamine receptors. Nature 1976;261:717.

34. Marder SR, van Kammen DP. Dopamine receptor antagonists. In: Kaplan H, Saddock B, eds. Comprehensive

Textbook of Psychiatry VII. New York: Lippincott Williams & Wilkins; 1999:2356.

35. Davidson M et al. L-Dopa challenge and relapse in schizophrenia. Am J Psychiatry 1987;144:934.

36. Lieberman J et al. Methylphenidate challenge as a predictor of relapse in schizophrenia. Am J Psychiatry

1984;141:633.

37. Snyder S. Catecholamines in the brain as mediators of amphetamine psychosis. Arch Gen Psychiatry

1972;27:169.

38. Littrel RA et al. The neurobiology of schizophrenia. Pharmacotherapy 1996;16:143S.

39. Risch SC. Pathophysiology of schizophrenia and the role of newer antipsychotics. Pharmacotherapy

1996;116(Suppl 1 Pt 2):11S.

40. Davis KL et al. Dopamine in schizophrenia: A review and reconceptualization. Am J Psychiatry

1991;148:1474.

P.78p31

Page 48 of 62Ovid: Applied Therapeutics: The Clinical Use Of Drugs

17/8/2010mk:@MSITStore:C:\Users\askari\Desktop\E%20Book\Applied\Applied%20Therapeuti...

Page 49: Schizophrenia

41. Joyce JN. The dopamine hypothesis of schizophrenia: Limbic interactions with serotonin and

norepinephrine. Psychopharmacology 1993;112:S16.

42. Olney JW et al. Glutamate receptor dysfunction and schizophrenia. Arch Gen Psychiatry 1995;52:998.

43. Kim JS et al. Low cerebrospinal fluid glutamate in schizophrenia patients and new hypothesis on

schizophrenia. Neurosci Lett 1980;20:379.

44. Akbarian S et al. Gene expression for glutamic acid decarboxylase is reduced without loss of neurons in

prefrontal cortex of schizophrenics. Arch Gen Psychiatry 1995;52:528.

45. Kraepelin E. Dementia praecox and paraphrenia. Trans. RM Barclay. Chicago: Chicago Medical Book; 1919.

46. Bleuler E. Dementia Praecox or the Group of Schizophrenias (1911). Trans. J Zinkin. New York:

International Press; 1950.

47. Schneider K. Clinical Psychopathology. New York: Grune & Stratton; 1959.

48. Thacore VR, Shukla SRP. Cannabis psychosis and paranoid schizophrenia. Arch Gen Psychiatry 1976;33:383.

49. Palmer BW et al. Is it possible to be schizophrenic and neuropsychologically impaired? Neuropsychology

1997;11:3437.

50. Green MF et al. Neurocognitive deficits and functional outcome in schizophrenia: are we measuring the

“right stuff”? Schizophr Bull 2000;26:119.

51. Beiser M et al. Establishing the onset of psychotic illness. Am J Psychiatry 1993;150:1349–1354.

52. McGorry PD et al. The prevalence of prodromal features of schizophrenia in adolescence: a preliminary

study. Acta Psychiatr Scand 1995:92:241–249.

53. Duzyurek S, Wienerm JM. Early recognition in schizophrenia: the prodromal stages. J Pract Psychiatr Behav

Health 1999;5:187–196.

54. Bleuler M. The Schizophrenic Disorders: Long-Term Patient and Family Studies. New Haven, CT: Yale

University Press; 1978.

55. Ciompi L. Catamnestic long-term study on the course of life and aging of schizophrenics. Schizophr Bull

1980;6:608.

56. Huber G et al. Longitudinal studies of schizophrenic patients. Schizophr Bull 1980;6:592.

57. McGlashan TH, Fenton WS. Subtype progression and pathophysiologic deterioration in early schizophrenia.

Schizophr Bull 1993;19:71.

58. American Psychiatric Association. Practice guideline for the treatment of patients with schizophrenia. 2nd

Page 49 of 62Ovid: Applied Therapeutics: The Clinical Use Of Drugs

17/8/2010mk:@MSITStore:C:\Users\askari\Desktop\E%20Book\Applied\Applied%20Therapeuti...

Page 50: Schizophrenia

ed. 2004. Available at http://www.psychiatryonline.com/pracGuideTopic_6.aspx.

59. Newman SC, Bland RC. Mortality in a cohort of patients with schizophrenia: a record linkage study. Can J

Psychiatry 1991;36:239.

60. Caldwell CB, Gottesman II. Schizophrenics kill themselves too: a review of risk factors for suicide.

Schizophr Bull 1990;16:571.

61. Wyatt RJ. Neuroleptics and the natural course of schizophrenia. Schizophr Bull 1992;17:325.

62. Robinson DG et al. Predictors of treatment response from a first episode of schizophrenia or schizoaffective

disorder. Am J Psychiatry 1999;156:544.

63. Rifkin A. Pharmacologic strategies in the treatment of schizophrenia. Psychiatr Clin North Am 1993;16:351.

64. Kane JM. Treatment programme and long term outcome in chronic schizophrenia. Acta Psychiatr Scand

1990;358(Suppl):151.

65. Davis JM. Overview: maintenance therapy in psychiatry. I: schizophrenia. Am J Psychiatry 1975;132:1237.

66. Janicak PG et al. Principles and Practice of Psychopharmacotherapy. Baltimore, MD: Williams & Wilkins;

1997:97.

67. Heinssen RK et al. Psychosocial skills training for schizophrenia: Lessons from the laboratory. Schizophr Bull

2000;26:21.

68. Beck AT, Rector NA. Cognitive therapy for schizophrenia patients. Harvard Ment Health Lett 1998;15:4.

69. Dixon L et al. Update on family psychoeducation for schizophrenia. Schizophr Bull 2000;26:5.

70. Cook JA, Razzano L. Vocational rehabilitation for persons with schizophrenia: Recent research and

implications for practice. Schizophr Bull 2000;26:87.

71. Garety PA et al. Cognitive-behavioral therapy for medication-resistant symptoms. Schizophr Bull 2000;

26:73.

72. Saddock BJ, Saddock VA. The doctor-patient relationship and interviewing techniques. In: Kaplan &

Sadock's Synopsis of Psychiatry: Behavioral Sciences, Clinical Psychiatry. Philadelphia: Lippincott Williams &

Wilkins; 2003:1.

73. Saddock BJ, Saddock VA. Psychiatric history and mental status examination. In: Kaplan & Sadock's Synopsis

of Psychiatry: Behavioral Sciences, Clinical Psychiatry. Philadelphia: Lippincott Williams & Wilkins; 2003:229.

74. Wolkowitz OM, Pickar D. Benzodiazepines in the treatment of schizophrenia: a review and reappraisal. Am

J Psychiatry 1991;148:714.

Page 50 of 62Ovid: Applied Therapeutics: The Clinical Use Of Drugs

17/8/2010mk:@MSITStore:C:\Users\askari\Desktop\E%20Book\Applied\Applied%20Therapeuti...

Page 51: Schizophrenia

75. Growe GA et al. Lithium in chronic schizophrenia. Am J Psychiatry 1979;136:454.

76. Small JG et al. A placebo-controlled study of lithium combined with neuroleptics in chronic schizophrenic

patients. Am J Psychiatry 1975;132:1315.

77. Goldney RD, Spence ND. Safety of the combination of lithium and neuroleptic drugs. Am J Psychiatry

1986;143:882.

78. Leucht S et al. Carbamazepine augmentation for schizophrenia: how good is the evidence? J Clin

Psychiatry. 2002;63:218.

79. McElroy SL et al. Sodium valproate: its use in primary psychiatric disorders. J Clin Psychopharmacol

1987;7:16.

80. Neppe VM. Carbamazepine in nonresponsive psychosis. J Clin Psychiatry 1988;49(Suppl 4):22.

81. Citrome L et al. Changes in use of valproate and other mood stabilizers for patients with schizophrenia

from 1994 to 1998. Psychiatr Serv 2000;51:634.

82. Casey DE et al. Effect of divalproex combined with olanzapine or risperidone in patients with an acute

exacerbation of schizophrenia. Neuropsychopharmacology. 2003;28:182.

83. Sorgi PJ et al. Beta-adrenergic blockers for the control of aggressive behaviors in patients with chronic

schizophrenia. Am J Psychiatry 1986;143:775.

84. Lindstrom L, Persson E. Propranolol in chronic schizophrenia: a controlled study in neuroleptic treated

patients. Br J Psychiatry 1980;137:126.

85. Perry PJ et al. Psychotropic Drug Handbook. Washington, DC: American Psychiatric Press; 1997.

86. Ames D et al. Advances in antipsychotic pharmacotherapy: clozapine, risperidone, and beyond. Essent

Psychopharmacol 1996;1:5.

87. Lohr JB, Braff DL. The value of referring to recently introduced antipsychotics as “second generation.” Am

J Psychiatry 2003;160:1371.

88. Nordstrom AL et al. Central D2-dopamine receptor occupancy in relation to antipsychotic drug effects: a

double-blind PET study of schizophrenic patients. Biol Psychiatr 1993;33:227.

89. Kapur S et al. Relationship between dopamine D2 receptor occupancy, clinical response, and side effects: a

double-blind PET study. Am J Psychiatry 2000;157:514.

90. Marder SR, Van Putten T. Antipsychotic medications. In: Schatzberg AF, Nemeroff CB, eds. The American

Psychiatric Press Textbook of Psychopharmacology. Washington, DC: American Psychiatric Press; 1995:247.

91. Bench CJ et al. The time course of binding to striatal dopamine D2 receptors by the neuroleptic ziprasidone

Page 51 of 62Ovid: Applied Therapeutics: The Clinical Use Of Drugs

17/8/2010mk:@MSITStore:C:\Users\askari\Desktop\E%20Book\Applied\Applied%20Therapeuti...

Page 52: Schizophrenia

(CP-88,059-01) determined by positron emission tomography. Psychopharmacology (Berl) 1996;124:141.

92. Kapur S et al. Clinical and theoretical implications of 5-HT2 and D2 receptor occupancy of clozapine,

risperidone and olanzapine in schizophrenia. Am J Psychiatry 1999;156:286.

93. Kapur S et al. A positron emission tomography study of quetiapine in schizophrenia: A preliminary finding of

an antipsychotic effect with only transiently high dopamine D2 receptor occupancy. Arch Gen Psychiatry

2000;57:553.

94. Meltzer HY. The role of serotonin in antipsychotic drug action. Neuropsychopharmacology 1999;21(2

Suppl):106S.

95. Nordstrom AL et al. D1, D2, and 5-HT2 receptor occupancy in relation to clozapine serum concentration: a

PET study of schizophrenic patients. Am J Psychiatry 1995;152:1444.

96. Fischman AJ et al. Positron emission tomographic analysis of central 5-hydroxytryptamine2 receptor

occupancy in healthy volunteers treated with the novel antipsychotic agent, ziprasidone. J Pharmacol Exp Ther

1996;279:939.

97. Stahl SM. Antipsychotic agents. In: Essential Psychopharmacology: Neuroscientific Basis and Practical

Applications. New York: Cambridge University Press; 2000;401.

98. Kane JM et al. Efficacy and safety of aripiprazole and haloperidol versus placebo in patients with

schizophrenia and schizoaffective disorder. J Clin Psychiatry. 2002;63:763.

99. Janssen LP. Paliperidone package insert. Titusville, NJ; February 2008.

100. Paliperidone (Invega) for schizophrenia. Med Lett Drugs Ther. 2007;49:21.

101. Casey DE. Side effect profiles of new antipsychotic agents. J Clin Psychiatry 1996;57(Suppl 11):40.

102. Lehman AF, Steinwachs DM. Translating research into practice: the Schizophrenia Patient Outcomes

Research Team (PORT) treatment recommendations. Schizophr Bull 1998;24:1.

103. Marder SR et al. The effects of risperidone on the five dimensions of schizophrenia derived by factor

analysis: combined results of the North American trials. J Clin Psychiatry 1997;58:538.

104. Hoyberg OJ et al. Risperidone versus perphenazine in the treatment of chronic schizophrenic patients with

acute exacerbations. Acta Psychiatr Scand 1993;88:395.

105. Tollefson GD et al. Olanzapine versus haloperidol in the treatment of schizophrenia and schizoaffective

disorder and schizophreniform disorders: results of an international collaborative trial. Am J Psychiatry

1997;154:457.

106. Beasley CM Jr et al. Olanzapine versus placebo and haloperidol: acute phase results of the North American

double-blind olanzapine trial. Neuropsychopharmacology 1996;14:111.

P.78p32

Page 52 of 62Ovid: Applied Therapeutics: The Clinical Use Of Drugs

17/8/2010mk:@MSITStore:C:\Users\askari\Desktop\E%20Book\Applied\Applied%20Therapeuti...

Page 53: Schizophrenia

107. Arvanitis LA et al. Multiple fixed doses of “Seroquel” (quetiapine) in patients with acute exacerbation of

schizophrenia: a comparison with haloperidol and placebo. Biol Psychiatr 1997;42:233.

108. Peuskens J, Link CG. A comparison of quetiapine and chlorpromazine in the treatment of schizophrenia.

Acta Psychiatr Scand 1997;96:265.

109. Goff DC et al. An exploratory haloperidol-controlled dose-finding study of ziprasidone in hospitalized

patients with schizophrenia or schizoaffective disorder. J Clin Psychopharmacol 1998;18:296.

110. Daniel DG et al. Ziprasidone 80 mg/day and 160 mg/day in the acute exacerbation of schizophrenia and

schizoaffective disorder: a 6-week placebo-controlled trial. Neuropsychopharmacology 1999;20:491.

111. Kane J et al. Treatment of schizophrenia with paliperidone extended-release tablets: a 6-week placebo-

controlled trial. Schizophr Res 2007;90:147.

112. Davidson M et al. Efficacy, safety and early response of paliperidone extended release tablets

(paliperidone ER): results of a 6-week, randomized, placebo-controlled study. Schizophr Res 2007;93:117.

113. Marder SR et al., in press. Efficacy and safety of paliperidone extended-release tablets: results of a 6-

week, randomized, placebo-controlled study. Biol Psychiatry.

114. Geddes J et al. Atypical antipsychotics in the treatment of schizophrenia: systematic overview and meta-

regression analysis. Br Med J 2000;321:1371.

115. Leucht S. Efficacy and extrapyramidal side-effects of the new antipsychotics olanzapine, quetiapine,

risperidone, and sertindole compared to conventional antipsychotics and placebo: a meta-analysis of

randomized controlled trials. Schizopr Res 1999;35:51.

116. Davis JM et al. A meta-analysis of the efficacy of second-generation antipsychotics. Arch Gen Psychiatry

2003;60:553.

117. Marder SR, Meibach RC. Risperidone in the treatment of schizophrenia Am J Psychiatry 1994;151:825.

118. Moller HJ. Neuroleptic treatment of negative symptoms in schizophrenic patients. Efficacy problems and

methodological difficulties. Eur Neuropsychopharmacol 1993;3:1.

119. Tollefson GD, Sanger TM. Negative symptoms: a path analytic approach to a double-blind, placebo- and

haloperidol-controlled clinical trial with olanzapine. Am J Psychiatry 1997;154:466.

120. Hollander E et al. A double-blind placebo-controlled pilot study of olanzapine in childhood/adolescent

pervasive developmental disorder. J Child Adolesc Psychopharmacol 2006;16:541.

121. Nasrallah HA. The Roles of Efficacy, Safety, and Tolerability in Antipsychotic Effectiveness: Practical

Implications of the CATIE Schizophrenia Trial. J Clin Psych 2007;68(Suppl 1):5.

122. Stroup TS, et al. Effectiveness of olanzapine, quetiapine and risperidone in patients with chronic

Page 53 of 62Ovid: Applied Therapeutics: The Clinical Use Of Drugs

17/8/2010mk:@MSITStore:C:\Users\askari\Desktop\E%20Book\Applied\Applied%20Therapeuti...

Page 54: Schizophrenia

schizophrenia after discontinuation of perphenazine: a CATIE Study. Am J Psych 2007;164:415.

123. Milton GV, Jann MW. Emergency treatment of psychotic symptoms: pharmacokinetic considerations for

antipsychotic drugs. Clin Pharmacokinet 1995;28:494.

124. Brook S et al. Intramuscular ziprasidone compared with intramuscular haloperidol in the treatment of

acute psychosis. J Clin Psychiatry 2000;61:933.

125. Daniel DG et al. Intramuscular ziprasidone 20 mg is effective in reducing agitation associated with

psychosis: a double-blind, randomized trial. Psychopharmacology 2001;155:128.

126. Battaglia J. Pharmacological management of acute agitation. Drugs 2005;65:1207.

127. Garza-Trevino ES et al. Efficacy of combinations of intramuscular antipsychotics and sedative-hypnotics

for control of psychotic agitation. Am J Psychiatry 1989;146:1598.

128. Baldessarini R et al. Significance of neuroleptic dose and plasma level in pharmacological treatment of

psychoses. Arch Gen Psychiatry 1988;45:79.

129. Hillard JR. Emergency treatment of acute psychosis. J Clin Psychiatry 1998;59(Suppl):57.

130. Altamura AC et al. Intramuscular preparations of antipsychotics: uses and relevance in clinical practice.

Drugs 2003;63:493.

131. Dubin WR. Rapid tranquilization: antipsychotics or benzodiazepines? J Clin Psychiatry 1988;49(Suppl):5.

132. Bristol Myers Squibb. Aripiprazole package insert. Princeton, NJ: May 2008.

133. Andrienza R et al. Intramuscular aripiprazole for the treatment of acute agitation in patients with

schizophrenia or schizoaffective disorder: a double-blind, placebo-controlled comparison with intramuscular

haloperidol. Psychopharmacology 2006;188:281.

134. Janssen LP. Risperidone package insert. Titusville, NJ: February 2008.

135. Currier GW et al. Acute treatment of psychotic agitation: a randomized comparison of oral treatment with

risperidone and lorazepam vs. intramuscular treatment with haloperidol and lorazepam. J Clin Psychiatry

2004;65:386.

136. Lesem MD et al. Intramuscular ziprasidone, 2 mg versus 10 mg, in the short-term management of agitated

patients. J Clin Psychiatry 2001;62:12.

137. Breier A et al. A double-blind, placebo-controlled dose-response comparison of intramuscular olanzapine

and haloperidol in the treatment of acute agitation in schizophrenia. Arch Gen Psychiatry 2002;59:441.

138. Pfizer. Ziprasidone package insert. New York: March 2007.

Page 54 of 62Ovid: Applied Therapeutics: The Clinical Use Of Drugs

17/8/2010mk:@MSITStore:C:\Users\askari\Desktop\E%20Book\Applied\Applied%20Therapeuti...

Page 55: Schizophrenia

139. Eli Lilly and Company. Olanzapine package insert. Indianapolis, IN: March 2008.

140. Ganguli R. Rationale and strategies for switching antipsychotics. Am J Health-Syst Pharm 2002;59:S22.

141. Kane JM et al. Long-acting injectable risperidone: efficacy and safety of the first long-acting atypical

antipsychotic. Am J Psychiatry 2003160: 1125.

142. Martin SD et al. Clinical experience with the long-acting injectable formulation of the atypical

antipsychotic, risperidone. Curr Med Res Opin 2003;19:298.

143. Burns MJ. The pharmacology and toxicology of atypical agents. J Clin Toxicol 2001;39:1.

144. Ereshefsky L. Pharmacokinetics and drug interactions: update for new antipsychotics. J Clin Psychiatry

1996;57(Suppl 11):12.

145. Michalets EL. Update: clinically significant cytochrome P450 drug interactions. Pharmacotherapy

1998;18:84.

146. Jann MW et al. Effects of carbamazepine on plasma haloperidol levels. J Clin Psychopharmacol

1985;5:106.

147. Rice DP. The economic impact of schizophrenia. J Clin Psychiatry 1999;60(Suppl 1):4.

148. Hudson TJ et al. Economic evaluations of novel antipsychotic medications: a literature review. Schizophr

Res 2003;60:199.

149. Casey DE. Neuroleptic-induced extrapyramidal syndromes and tardive dyskinesia. In: Hirsch SR,

Weinberger DR, eds. Schizophrenia. Oxford, UK: Blackwell; 1995:546.

150. Casey DE. Tardive dyskinesia and atypical antipsychotic drugs. Schizophr Res 1999;35(Suppl):55–62.

151. Dolder CR, Jeste DV. Incidence of tardive dyskinesia with typical versus atypical antipsychotics in very

high risk patients. Biol Psychiatry 2003;53:1142.

152. Beasley CM et al. Randomized double-blind comparison of the incidence of tardive dyskinesia in patients

with schizophrenia during long-term treatment with olanzapine or haloperidol. Br J Psychiatry 1999;174:23.

153. Jeste DV et al. Lower incidence of tardive dyskinesia with risperidone compared with haloperidol in older

patients. J Am Geriatr Soc 1999;47:716.

154. Holloman LC, Marder SR. Management of acute extrapyramidal effects induced by antipsychotic drugs. Am

J Health-Syst Pharm 1997;54:2461.

155. Jeste DV, Naimark D. Medication-induced movement disorders. In: Tasman A, Kay J, Lieberman J, eds.

Psychiatry. Philadelphia: WB Saunders; 1996: 1304.

Page 55 of 62Ovid: Applied Therapeutics: The Clinical Use Of Drugs

17/8/2010mk:@MSITStore:C:\Users\askari\Desktop\E%20Book\Applied\Applied%20Therapeuti...

Page 56: Schizophrenia

156. Simpson G, Angus JWS. A rating scale for extrapyramidal side effects. Acta Psychiatr Scand 1970;46(Suppl

221):11.

157. World Health Organization. Prophylactic use of anticholinergics in patients on long-term neuroleptic

treatment: a consensus statement. World Health Organization heads of centres collaborating on WHO co-

ordinated studies on biological aspects of mental illness. Br J Psychiatry 1990;156:412.

158. Shirzadi AA and Ghaemi SN. Side effects of atypical antipsychotics: extrapyramidal symptoms and the

metabolic syndrome. Harv Rev Psych 2006;14:152.

159. Fleischhacker WW et al. The pharmacological treatment of neuroleptic-induced akathisia. J Clin

Psychopharmacol 1990;10:12.

160. Kane JM et al. Tardive dyskinesia: prevalence, incidence, and risk factors. J Clin Psychopharmacol 1988;8

(4 Suppl):52S.

161. Kane JM et al. Tardive Dyskinesia: A Task Force Report to the American Psychiatric Association.

Washington, DC: American Psychiatric Association; 1992.

162. Jeste DV et al. Risk of tardive dyskinesia in older patients. A prospective longitudinal study of 266

outpatients. Arch Gen Psychiatry 1995;52:756.

163. Woerner MG et al. Prospective study of tardive dyskinesia in the elderly: rates and risk factors. Am J

Psychiatry 1998;155:1521.

164. Yassa R. Functional impairment in tardive dyskinesia: medical and psychosocial dimensions. Acta Psychiatr

Scand 1989;80:64.

165. Lantz MS. Tardive dyskinesia: an antipsychotic side effect that has not gone away. Clin Geriatr

2005;13:18.

166. McEvoy JP et al. The expert consensus guideline series. Treatment of Schizophrenia, 1999. J Clin

Psychiatry 1999;60(Suppl 11):1.

167. Boomershine KH et al. Vitamin E in the treatment of tardive dyskinesia. Ann Pharmacother 1999;33:1195.

168. Adler LA et al. Vitamin E treatment for tardive dyskinesia. Arch Gen Psychiatry 1999;56: 836.

169. Kane JM et al. Does clozapine cause tardive dyskinesia? J Clin Psychiatry 1993;54:327.

170. Janicak PG et al. Principles and Practice of Psychopharmacotherapy. Baltimore, MD: Williams & Wilkins;

1997:188.

171. Rogers DP, Shramko JK. Therapeutic options in the treatment of clozapine-induced sialorrhea.

Pharmacotherapy. 2000;20:1092.

Page 56 of 62Ovid: Applied Therapeutics: The Clinical Use Of Drugs

17/8/2010mk:@MSITStore:C:\Users\askari\Desktop\E%20Book\Applied\Applied%20Therapeuti...

Page 57: Schizophrenia

172. Buckley NA, Sanders P. Cardiovascular adverse effects of antipsychotic drugs. Drug Saf 2000;23:215.

173. FDA Psychopharmacological Drugs Advisory Committee. 19 July 2000 Briefing Document for Ziprasidone

HCl. Available at http://www.fda.gov/ohrms/dockets/ac/00/backgrd/3619b1a.pdf.

174. Petty R. Prolactin and antipsychotic medications: mechanism of action. Schizophr Res 1999;35:S67.

175. Wirshing DA et al. Understanding the new and evolving profile of adverse drug effects in schizophrenia.

Psychiatr Clin North Am 2003;26:165.

176. Kleinberg DR et al. Prolactin levels and adverse events in patients treated with risperidone J Clin

Psychopharmacol 1999;19:57.

177. Dickson RA, Glazer WM. Neuroleptic-induced hyperprolactinemia. Schizophren Res 1999;35:S75.

178. McIntyre RS et al. Mechanisms of antipsychotic-induced weight gain. J Clin Psychiatry 2001;62(Suppl

23):23.

179. Goff DC et al. Medical Morbidity and Mortality in Schizophrenia: Guidelines for Psychiatrists. J Clin Psych

2005;66:183.

180. Nasrallah HA. Atypical antipsychotic-induced metabolic side effects: insights from receptor-binding

profiles. Mol Psych 2008;13:27.

181. Nasrallah HA, Newcomer JW. Atypical antipsychotics and metabolic dysregulation: evaluating the

risk/benefit equation and improving the standard of care. J Clin Psychopharm 2004;24:S7.

182. Tandon R. Safety and tolerability: how do newer generation “atypical” antipsychotics compare? Psychiatr

Q 2002;73:297.

183. Allison DB et al. Antipsychotic-induced weight gain: a comprehensive research synthesis. Am J Psychiatry

1999;156:1686.

184. Sussman N. Review of atypical antipsychotics and weight gain. J Clin Psychiatry 2001;62(Suppl 23):5.

185. Henderson D et al. Clozapine: Diabetes mellitus, weight gain and lipid abnormalities: a five year

naturalistic study. Am J Psychiatry 2000;157:975.

186. Kinon BJ et al. Long-term olanzapine treatment: weight change and weight-related health factors in

schizophrenia J Clin Psychiatry 2001;62:92.

187. McIntyre RS et al. Antipsychotic metabolic effects: Weight gain, diabetes mellitus, and lipid

abnormalities. Can J Psychiatry 2001;46:273.

188. Fontaine KR et al. Estimating the consequences of antipsychotic induced weight gain on health and

mortality rate. Psychiatry Res 2001;101:277.

P.78p33

Page 57 of 62Ovid: Applied Therapeutics: The Clinical Use Of Drugs

17/8/2010mk:@MSITStore:C:\Users\askari\Desktop\E%20Book\Applied\Applied%20Therapeuti...

Page 58: Schizophrenia

189. Goldstein LE, Henderson DC. Atypical antipsychotics agents and diabetes mellitus. Prim Psychiatry

2000;7:65.

190. Melkersson KI et al. Elevated levels of insulin, leptin, and blood lipids in olanzapine-treated patients with

schizophrenia or related psychoses. J Clin Psychiatry 2000;61:742.

191. Newcomer JW. Metabolic considerations in the use of antipsychotic medications: a review of recent

evidence. J Clin Psych 2007;68(Suppl 1):20.

192. Garattini S et al. Reduction of food intake by manipulation of central serotonin: current experimental

results. Br J Psychiatry 1989;155:41.

193. Must A et al. The disease burden associated with overweight and obesity. JAMA 1999;282:1523.

194. Meyer JM. Strategies for the long-term treatment of schizophrenia: real world lessons from the CATIE

trial. J Clin Psych 2007;68:28.

195. Consensus Development Conference on Antipsychotic Drugs and Obesity and Diabetes. Diabetes Care

2004;27:596.

196. Treating Schizophrenia: A Quick Reference Guide. American Psychiatric Association Website. Available at:

http://www.psych.org.

197. Farver DK. Neuroleptic malignant syndrome induced by atypical antipsychotics. Expert Opin Drug Saf

2003;2:21.

198. Ozen ME et al. Neuroleptic malignant syndrome induced by ziprasidone on the second day of treatment.

World J Biol Psychiatry 2007;8:42.

199. Evcimen H et al. Neuroleptic malignant syndrome induced by low dose aripiprazole in first episode of

psychosis. J Psychiatr Pract 2007;13:117.

200. Liebold J et al. Neuroleptic malignant syndrome associated with ziprasidone in an adolescent. Clin Ther

2004;26:1105.

201. Molina D et al. Aripiprazole as the causative agent of neuroleptic malignant syndrome: a case report.

Prime Care Companion J Clin Psychiatry 2007;9:148.

202. Borovicka MC et al. Ziprasidone and lithium-induced neuroleptic malignant syndrome. Ann Pharmacother

2006;40:139.

203. Kang SG et al. Atypical neuroleptic malignant syndrome associated with aripiprazole. J Clin

Psychopharmacol 2006; 26:534.

204. Caroff SN et al. Neuroleptic malignant syndrome. Med Clin North Am 1993;77:185.

Page 58 of 62Ovid: Applied Therapeutics: The Clinical Use Of Drugs

17/8/2010mk:@MSITStore:C:\Users\askari\Desktop\E%20Book\Applied\Applied%20Therapeuti...

Page 59: Schizophrenia

205. Pelonero AL et al. Neuroleptic malignant syndrome: a review. Psychiatr Serv 1998;49:1163.

206. Lazarus A. Therapy of neuroleptic malignant syndrome. Psychiatr Dev 1986;4:19.

207. Rosebush PI et al. Twenty neuroleptic rechallenges after neuroleptic malignant syndrome in 15 patients. J

Clin Psychiatry 1989;50:295.

208. AstraZeneca Pharmaceuticals. Quetiapine Package Insert. Wilmington, DE: May 2008.

209. Simpson CM et al. Adverse effects of antipsychotic drugs. Drugs 1981;21:138.

210. Devinsky O et al. Clozapine-related seizures. Neurology 1991;41:369.

211. Hummer M, Fleichhacker WW. Nonmotor side effects of novel antipsychotics. Curr Opin CPNS Invest Drugs

2000;2:45.

212. Kotin J et al. Thioridazine and sexual dysfunction. Am J Psychiatry 1976;133:82.

213. Meston CM, Frohlich PF. The neurobiology of sexual function. Arch Gen Psychiatry 2000;57:1012.

214. Aizenberg D et al. Sexual dysfunction in male schizophrenic patients. J Clin Psychiatry 1995;56:137.

215. Mossum D. A decision analysis approach to neuroleptic dosing: insights from a mathematical model. J Clin

Psychiatry 1997;58:66.

216. Conley RR, Kelly DL. Management of treatment resistance in schizophrenia. Biol Psychiatry 2001;50:898.

217. Woods SW. Chlorpromazine equivalent doses for the newer atypical antipsychotics. J Clin Psychiatry

2003;64:663.

218. Ereshefsky L et al. Future of depot neuroleptic therapy: pharmacokinetic and pharmacodynamic

approaches. J Clin Psychiatry 1984;45:50.

219. McEvoy GK, ed. American Hospital Formulary System Drug Information. Bethesda, MD: American Society

of Hospital Pharmacists; 1998:1864.

220. McNeil Pharmaceutical. Haldol decanoate package insert. Raritan, NJ: September 2001.

221. Meltzer HY. New drugs for the treatment of schizophrenia. Psychiatry Clin North Am 1993;16:365.

222. Kane J et al. Clozapine for treatment resistant schizophrenia. A double blind comparison with

chlorpromazine. Arch Gen Psychiatry 1988;45:789.

223. Chakos M et al. Effectiveness of second-generation antipsychotics in patients with treatment-resistant

Page 59 of 62Ovid: Applied Therapeutics: The Clinical Use Of Drugs

17/8/2010mk:@MSITStore:C:\Users\askari\Desktop\E%20Book\Applied\Applied%20Therapeuti...

Page 60: Schizophrenia

schizophrenia: a review and meta-analysis of randomized trials. Am J Psychiatry 2001;158:518.

224. Wirshing DA et al. Risperidone in treatment-refractory schizophrenia. Am J Psychiatry 1999;156:1374.

225. Conley RR et al. Olanzapine compared with chlorpromazine in treatment-resistant schizophrenia. Am J

Psychiatry 1998;155:914.

226. Pantelis C, Barnes TR. Drug strategies and treatment-resistant schizophrenia. Aust N Z J Psychiatry

1996;30:20.

227. Miller AL et al. The TMAP schizophrenia algorithms. J Clin Psychiatry 1999;60:649.

228. Breier A et al. Clozapine treatment of outpatients: outcome and long-term response patterns. Hosp Comm

Psychiatry 1993;44:1145.

229. Meltzer HY. Duration of a clozapine trial in neuroleptic-resistant schizophrenia. Arch Gen Psychiatry

1989;46:672.

230. Meltzer HY. Treatment of the neuroleptic-nonresponsive schizophrenic patient. Schizophr Bull

1992;18:515.

231. Novartis Pharmaceuticals. Clozapine package insert. East Hanover, NJ: May 2008.

232. Freudenreich O et al. Clozapine-induced sialorrhea treated with sublingual ipratropium spray: a case

series. J Clin Psychopharmacol 2004;24:98.

233. Gaftanyuk O, Trestman RL. Scopolamine patch for clozapine-induced sialorrhea. Psych Serv 2004;55:318.

234. Webber MA, et al. Guanfacine treatment of clozapine-induced sialorrhea. J Clin Psychopharmacol

2004;24:675.

235. Kahl KG et al. Botulinum toxin as an effective treatment of clozapine-induced sialorrhea treatment.

Psychopharmacology 2004;173:229.

236. Sharma A et al. Intraoral application of atropine sulfate ophthalmic solution of clozapine-induced

sialorrhea. Ann Pharmacother 2004;38:1538.

237. Praharaj SK et al. Clozapine-induced sialorrhea: pathophysiology and management strategies.

Psychopharmacology 2006;185:265.

238. Alvir JMJ et al. Clozapine-induced agranulocytosis: incidence and risk factors in the United States. N Engl

J Med 1993;329:162.

239. Gerson SL. G-CSF and the management of clozapine-induced agranulocytosis. J Clin Psychiatry 1994;55

(Suppl 9B):139.

240. Preskorn SH et al. Therapeutic drug monitoring: principles and practice. Psychiatr Clin North Am

Page 60 of 62Ovid: Applied Therapeutics: The Clinical Use Of Drugs

17/8/2010mk:@MSITStore:C:\Users\askari\Desktop\E%20Book\Applied\Applied%20Therapeuti...

Page 61: Schizophrenia

1993;16:611.

241. Volavka J et al. Plasma haloperidol levels and clinical effects in schizophrenia and schizoaffective

disorder. Arch Gen Psychiatry 1995;52:837.

242. Van Putten T et al. Haloperidol plasma levels and clinical response: a therapeutic window relationship.

Am J Psychiatry 1992;149:500.

243. Levison DF et al. Fluphenazine plasma levels, dosage, efficacy, and side effects. Am J Psychiatry

1995;152:765.

244. Mavroides M et al. Clinical relevance of thiothixene plasma levels. J Clin Psychopharmacol 1984;4:155.

245. Yesavage J et al. Correlation of initial thiothixene serum levels and clinical response. Arch Gen Psychiatry

1983;40:301.

246. Perry PJ et al. Clozapine and norclozapine plasma concentration and clinical response of treatment-

refractory schizophrenic patients. Am J Psychiatry 1991;148:231.

247. Freeman DJ et al. Will routine therapeutic drug monitoring have a place in clozapine therapy? Clin

Pharmacokinet 1997;32:93.

248. Perry PJ et al. Olanzapine plasma concentrations and clinical response: acute phase results of the North

American Olanzapine Trial. J Clin Psychopharmacol 2001;21:14.

249. Fenton WS et al. Determinants of medication compliance in schizophrenia: empirical and clinical findings.

Schizophr Bull 1997;23:637.

250. Lacro JP et al. Prevalence of and risk factors for medication nonadherence in patients with schizophrenia:

a comprehensive review of recent literature. J Clin Psychiatry 2002;63:892.

251. Dolder CR et al. Antipsychotic medication adherence: is there a difference between typical and atypical

agents? Am J Psychiatry 2002;159:103–.

252. Rosenheck R et al. Medication continuation and compliance: a comparison of patients treated with

clozapine and haloperidol. J Clin Psychiatry 2000;61:382.

253. Weiden PJ, Olfson M. Cost of relapse in schizophrenia. Schizophr Bull 1995;21:419.

254. Terkelsen KG, Menikoff A. Measuring the costs of schizophrenia: implications for the post-institutional era

in the U.S. Pharmacoeconomics 1995;52:173.

255. Barnes TR, Curson DA. Long-term depot antipsychotics. A risk-benefit assessment. Drug Saf 1994;10:464.

256. Altshuler LL et al. Pharmacologic management of psychiatric illness during pregnancy: dilemmas and

guidelines. Am J Psychiatry 1996;153:592.

Page 61 of 62Ovid: Applied Therapeutics: The Clinical Use Of Drugs

17/8/2010mk:@MSITStore:C:\Users\askari\Desktop\E%20Book\Applied\Applied%20Therapeuti...

Page 62: Schizophrenia

257. McCracken JT et al for Research Units on Pediatric Psychopharmacology (RUPP) Autism Network.

Risperidone in children with autism and serious behavioral problems. N Engl J Med 2002;347:314.

258. Troost PW et al. Long-term effects of risperidone in children with autism spectrum disorders: a placebo

discontinuation study. J Am Acad Child Adolesc Psychiatry 2005;44:1137.

259. Kemner C et al. Open-label study of olanzapine in children with pervasive developmental disorder. J Clin

Psychopharmacol 2002;22:455.

260. Malone RP et al. Olanzapine versus haloperidol in children with autistic disorder: an open-pilot study. J

Am Acad Child Adolesc Psychiatry 2001;40:887.

261. Roychoudhury K, Demb HB. Use of olanzapine in children with developmental disabilities and challenging

behaviors. J Investig Med 1999;47:65a.

262. Potenza MN et al. Olanzapine treatment of children, adolescents, and adults with pervasive

developmental disorders: an open-label pilot study. J Clin Psychopharmacol 1999;19:37.

263. Martin A et al. Open-label quetiapine in the treatment of children and adolescents with autistic disorder.

J Child Adolesc Psychopharmacol 1999;9:99.

264. McDougle CJ et al. Case series: use of ziprasidone for maladaptive symptoms in youths with autism. J Am

Acad Child Adolesc Psychiatry 2002;41:921.

265. Valicenti-McDermott MR, Demb H. Clinical effects and adverse reactions to off-label use of aripiprazole in

children and adolescents with developmental disabilities. J Child Adolesc Psychopharmacol 2006;16:549.

266. Marken PA, Stanislav SW. Schizophrenia. In: Koda-Kimble MA, Young LY, eds. Applied Therapeutics: The

Clinical Use of Drugs. Baltimore, MD: Lippincott Williams & Wilkins; 2001;76.

P.78p34

Page 62 of 62Ovid: Applied Therapeutics: The Clinical Use Of Drugs

17/8/2010mk:@MSITStore:C:\Users\askari\Desktop\E%20Book\Applied\Applied%20Therapeuti...